Anda di halaman 1dari 55

TES FORMATIF MODUL 2  BAHASA INDONESIA

1. Tujuan pembelajaran sastra anak di sekolah dasar adalah sebagai berikut , kecuali ….
a.  memberi kesenangan dan kebahagiaan 
b. mengembangkan kesadaran bersastra
c. mengembangkan pikiran negatif 
d. mengembangkan imajinasi anak
e. mengembangkan pengetahuan

2. Unsur bahasa itu terbatas, tetapi dengan unsur terbatas itu dapat dibuat satuan bahasa
yang tidak terbatas walaupun bersifat walaupun bersifat relatif, sesuai dengan sistem yang
berlaku dalam bahasa itu . contoh dari huruf p, a , I ,u dapat dibentuk kata palu, lupa dan
pula . kenyataan tersebut menunjukan bahwa bahasa itu bersifat ….
a. konvensional
b. unik
c. arbitrer
d. bervariasi
e. produktif

3. Pahami kalimat berikut ini ! kami sedang mendengarkan nasihat Pak Guru. kakak membeli
lima pack mineral untuk acara arisan di rumah. huruf yang bercetak miring pada kalimat
tersebut memiliki hubungan makna ….
a. homofon 
b. semua salah
c. homonim
d. homograf
e.  polisemi

4. Kelebihan pendekatan whole languange tertera di bawah ini, kecuali ….


a. penyampaian menyeluruh dan melibatkan berbagai disiplin ilmu
b. melibatkan berbagai komponen keterampilan berbahasa
c. melibatkan berbagai komponen kebahasaan 
d. melibatkan berbagai pakar kebudayaan
e. melibatkan lingkungan dan pengalaman nyata yang di alami anak-anak 

5. Orang-orang di Indonesia menyebut benda ini sebagai “buku” , padahal tidak ada hubungan
penalaran antara istilah “buku” dan rupa benda tersebut . Meski demikian , orang Indonesia
tetap sepakat untuk menyebut benda tersebut sebagai “buku” bukan “bola” . peristiwa ini
menunjukan bahwa …
a. bahasa itu unik dan konvensional
b. bahasa itu produktif dan arbitrer
c. bahasa itu bunyi dan bermakna
d. bahasa itu arbitrer dan konvensional
e. bahasa itu arbitrer dan bermakna 

6. Pendekatan dalam pembelajaran menulis yang penekanannya bergeser dari produk pada
proses apa yang dipikir dan ditulis siswa merupakan pendekatan ..
a. writting process
b. pendekatan komunikatif
c. pendekatan saintifik 
d. pendekatan konstruktivisme
e. pendekatan keterampilan proses 

7. “suami istri itu sedang berbelanja” kata-kata yang bercetakmiring pada kalimat di atas
tergolong frase..
a. eksosentrik atributif
b. eksosentrik
c. endosentrik kordinatif
d. endosentrik atributif
e. endosintrik apositif
8. Pengajaran apresiasi sastra yang mengarahkan siswa pada kegiatan menuangkan ide ,
gagasan dan perasaan imajinatif merupakan ….
a. apresiasi sastra kuratif
b. apresiasi sastra reseptif
c. apresiasi sastra ekspresif
d. apresiasi sastra fantastic
e. apresiasi sastra represif

9. Kehidupan itu seperti pedati yang berputar . Pada suatu saat , seseorang berasa di atas dan
pada saat lain kita berada di bawah. Ketika di atas hidup itu terasa enak dan
menyenangkan. Ketika di bawah hidup itu terasa sesak dan menyedihkan . Paragraf di atas
di kembangkan dengan pola ….
a. sebab akibat
b. analogi
c. perbandingan
d. penegasan 
e. pemberian contoh 

10. Bagi anak-anak SD , sastra memberikan sumbangan positif untuk proses perkembangan
anak, di bawah ini manfaat sastra untuk anak-anak SD adalah ….
a.  anak akan hafal teori dan sastra 
b. sastra menghambatperkembangan bahasa anak
c. lewat karya sastra kognisi siswa menjadi lambat
d. sastra anak dapt di manfaatkan untuk menanam nilai-nilai etik dalam kehidupan sehari-
hari
e. sastra bermanfaat untuk melatih perkembangan kepribadian negatif anak

11. Pernyataan berikut ini yang termasuk ragam bahasa dalam keputusan resmi adalah …
a. yang bertanda tangan di bawah ini kedua belah pihak dan sepakat menetapkan hal-hal
sebagai berikut
b. berdasarkan surat keputusan dekan nomor 15/UP/J36.1.1/2003, setiap kegiatan
mahasiswa harus di ketahui oleh dosen kurikulum .
c. ketua panitia lomba menerangkan bahwa nama-nama mahasiswi berikut ini telah
melaksanakan tugas sebagai juri .
d. Direktur bank buana cipta menetapkan nama-nama berikut ini sebagai panitia lelang
untuk tahun 2007 .
e. berkenaan dengan hasil rapat ketua organisasi mahasiswa,  kami sampaikan putusan
hasil rapat sebagai hasil rapat.   

12. Dalam pembelajaran SD , guru langsung menggunakan bahasa pengangtar Bahasa


Indonesia sedangkan muridnya dari berbagai latar belakang bahasa Ibu . Guru tersebut
menggunakan metode pembelajaran ….
a. langsung 
b. alamiah
c. terjemahan
d. tidak langsung
e. tata bahasa

13. Susunan kalimat “Apa kabar?” tidak bisa diubah menjadi “kabar apa?” . hal itu
membuktikan bahwa ….
a. bahasa itu bermakna
b. bahasa itu sistem
c. bahasa itu produktif
d. bahasa itu lambang
e. bahasa itu konvensional

14. Salah satu metode pembelajaran di sekolah dasar yang menuntut siswa untuk mengenal
dan dapat menggunakan ragam bahasa yang sesuai dengan yang diperankan adalah
metode …..
a. percakapan
b. wawancara
c. bercerita
d. bermain peran
e. reka cerita gambar

15. Ciri ragam semi ilmiah adalah sebagai beikut, kecuali …


a. kalimatnya efektif dan baku
b. lugas
c. situasinya semi formal
d. obyektif
e. gaya penyajiannya populer .

16. Sastra anak meliputi semua jenis penulisan kreatif dan imajinatif yang khususuntuk dibaca
dan menghibur anak—anak sehingga bahasa yang digunakan harus diperhatikan. Bahasa
yang digunakan sebaiknya memperhatikan hal berikut ini, kecuali ….
a. menghadirkan pemahaman dan pengalamantertentu pada anak
b. perkembangan globalisasi dunia dan perkembangan bahasa gaul 
c. perasaan dan pengalaman anak melalui pandangan anak-anak
d. menghadirkan imajeri (daya bayang)
e. perkembangan intelektual dan emosional anak

17. Ciri pendekatan whole languange tertera di bawah ini , kecuali ….


a. konseptual
b. bermakna
c. fungsional
d. menyeluruh 
e. lintas bidang studi

18. Pendekatan yang bertujuan untuk mengembangkan kemampuan menggunakan bahasa


untuk berkomunikasi ..
a. pendekatan tematik integrative
b. pendekatan integrated
c. pendekatan komunikatif
d. pendekatan kontruksivisme
e. pendekatan whole language

19. Di indonesia orang bisa menyebut “air” untuk menunjukan pada benda yang bersifat cair ,
bisa direbus, bisa dipakai untuk mandi dan mencuci. Namun di inggris orang menyebutnya
“water” sedangakn di saudi arabia “ma’an”. orang jawa punya sebutan lain lagi yakni
“banyu”. sementara orang sunda menyebutnya “cai”. hal itu membuktikan bahwa…
a. bahasa itu relative
b. bahasa itu unik
c. bahasa itu lambang
d. bahasa itu produktif
e. bahasa itu arbitrer

20. Anak mampu memahami struktur cerita secara simbolik melalui bahasa, permainan dan
gambar, demikian pula anak memahami alur atau hubungan yang ada dalam cerita .cir-ciri
tersebut menggambarkan perkembangan sastra anak pada usia….
a. usia 1-2 tahun
b.usia7 – 11 tahun
c.usia 11 – 13 tahun
d. usia 17 – 18 tahun
e. usia 2 – 7 tahun

21. Penulisan kata depan di yang benar terdapat pada kalimat ….


a. Ani duduk disamping Wati.
b. Kota Medan berada diantara kota Aceh dan Padang
c. Kucing itu tidur dibawah meja
d. Dimana kita akan bertemu ?
e. kami membaca buku diperpustakaan 
22. Dapat mengetahui karakter orang lain, bakat, emosi dan kadar intelektualitas merupakan
salah satu fungsi bahasa sebagai ….
a. sebagai sarana komunikasi
b. sebagai sarana dalam memahami orang lain
c. sebagai sarana adaptasi dan integrasi
d. sebagai alat kontrol sosial 
e. sebagai alat memahami diri

23. Guru menempelkan gambar berseri di papan tulis. Siswa menuliskan kalimat di bawah
gambar . Setiap siswa membaca secara bergiliran kalimat yang telah dituliskan .
Kesalahandalam kalimat saat itu didiskusikan , siswa menuliskan kembali dalam bukunya
kalimat yang benar . Fokus kegiatan pembelajaran di atas adalah …..
a. menyimak
b. berbicara
c. menjelaskan 
d. membaca
e.  menulis

24. Keterampilan yang dituntut dalam membaca nyaring adalah ….


a. menguasai audiensnya 
b. memperhatikan kenyaringan sesuai dengan jumlah peserta
c. menguasai tanda-tanda baca
d. semua benar
e. memperhatikan cara berbicara sesuai dengan isi pembicaraan

25. Jenis sastra yang harus di ajarkan pada siswa SD tercantum di bawah ini, kecuali ….
a. cerita/dongeng
b. drama anak
c. puisi lama dan puisi baru
d. puisi anak
e. roman

26. Salah satu metode membaca permulaan yang berfokus pada pengenalan kata bermakna,
fungsional, bertahap , dan kontekstual adalah ….
a. metode kalimat
b. metode kata
c. metode suku kata
d. metode eja
e. metode steinberg

27. Salah satu pembelajaran untuk meningkatkan daya simak, siswa diminta untuk menyimak
suatu petunjuk. setelah menyimak, siswa disuruh menuliskannya sesuai dengan apa yang
disimak . Contoh kegiatan pembelajaran menyimak di atas merupakan menyimak ….
a. konversatif
b. apresiatif
c. eksplorasif
d. konsentratif

28. Buku bergambar berperan sangat penting dalam pembelajaran sastra anak terutama untuk
anak-anak TK dan SD kelas awal . Di bawah ini bukan merupakan pentingnya buku
bergambar dalam pembelajaran sastra anak adalah ….
a. melalui gambar siswa di mudahkan untuk mengenal tulisan yang dapat dibaca .
b. melalui gambar siswa mengalami kesulitan memahami alur cerita 
c. gambar berfungsi untuk mengilustrasikan : peonokhan , latar, dan kejadian yang dipakai
untuk membangun alur (plot)
d. dengan gambar memunculkan imajinasi dan imajineri pada diri siswa
e. melalui gambar siswa terdorong untuk mengapresiasi dan memiliki kecintaan terhadap
buku .
29. Dalam naskah sumpah pemuda pada urutan ketiga, bahasa indonesia di rumuskan
sebagai….
a. identitas nasional
b. bahasa pemersatu
c. lambang kebangsaan
d. bahasa negara
e. bahasa nasional

30. Ujaran manusia dapat dikatakan sebagai bahasa apabila ujaran tersebut mengandung ….
a. intonasi
b. jeda
c. semua salah
d. nada
e. makna

31. Salah satu metode pembelajaran berbicara di sekolah dasar yang menuntut siswa untuk
mengenal dan dapat menggunakan ragam bahasa yang sesuai dengan yang di perankannya
adalah metode …
a. bermain peran
b. wawancara
c.  percakapan 
d. reka cerita gambar
e. bercerita
SOAL PEDAGOGIK
1. Pernyataan berikut yang menjelaskan makna istilah kognitif adalah…. 
A.kemampuan berkomunikasi
B. Kemampuan untuk memecahkan masalah 
C. kemampuan berinteraksi 
D.kemampuan untuk mengintegrasikan diri 

2. Perkembangan kognitif merupakan salah satu aspek yang sangat penting dalam
perkembangan peserta didik. Faktor utama yang mempengaruhi perkembangan kognitif
anak adalah..... 
A. inspirasi
B. budaya
C. inspirasi
D. pengasuhan dan lingkungan 

3. Kemampuan berfikir untuk mengoperasikan kaidah-kaidah logika tapi masih terkait


dengan obyek-obyek bersifat konkrit merupakan ciri-ciri kemampuan anak berusia
A.0 - 2 tahun
B.2 -- 7 tahun
C.7 -- 11/12 tahun
D.11/12/ -- 14/15 tahun 

4. Menggunakan potongan sapu lidi, kelereng, globe, gambar-gambar yang menyangkut


pembelajaran IPA serta IPS sebagai media adalah sesuai dengan tahapan perkembangan
berfikir anak yang dikenal sebagai tahapan
A. Anak memahami bilangan dan angka tetapi masih terkait dengan obyek bersifat
kongkrit (operasional konkrit)
B. Pengamatan dan penginderaan yang intensif terhadap lingkunganya (sensomotor)
C. Dominasi pengamatan bersifat egosentris
D. Kemampuan mengoperasikan kaidah logika yang tidak terikat lagi dengan obyek yang
bersifat konkrit (operasional formal) 
SOAL KOMPETENSI PEDAGOGIK PERKEMBANGAN SOSIAL-EMOSIONAL
PESERTA DIDIK  

5. Kemampuan peserta didik untuk membina hubungan dan kemampuan memotivasi diri
termasuk kecerdasan….
A. Kognitif
B. Sosial
C. Emosional
D. Moral 

6. Seorang peserta mampu mengendalikan diri dengan baik dalam berbagai situasi dan
mampu mampu menjalin kerjasama yang baik dengan teman-temannya, dan mampu
memposisikan diri di lingkungan dengan baik.
Peserta didik tersebut memiliki kecerdasan…
A. Moral
B. spiritual
C. Sosial emosional
D. Kognitif 

7. Seorang peserta didik selalu ingin mendominasi dalam suatu kelompok belajar. Dia tidak
memberi kesmpatan anggota lain untuk mengemukakan pendapat. Jika teman lain yang
memimpin dan mengendalikan jalannya diskusi, ia memisahkan diri dan cenderung
belajar sendiri. 
Peserta didik tersebut mengalami permasalahan dalam perkembangan 
A. sosial-emosional 
B. kognitif 
C. moral
D  spritual 
SOAL PERKEMBANGAN MORAL PESERTA DIDIK  

8. Peserta didik telah memiliki yang memiliki moralitas benar-benar diinternalisasikan dan
tidak didasarkan pada standar-standar orang lain. Dia mengenal tindakan moral alternatif,
menjajaki pilihan-pilihan, dan kemudian memutuskan berdasarkan suatu kode moral
pribadi.
Hal ini merupakan contoh perilaku moral-spritual pada tahapan ...
a.  penalaran pascakonvensional
b.  penalaran konvensional
c.  penalaran prakonvensional
d.   penalaran interkonvensional 

9. Individu memandang apa yang diharapkan oleh keluarga, kelompok, masyarakat dan
bangsa serta setia mendukung aturan social bukan hanya untuk ketenangan tetapi disadari
sebagai sesuatu yang berharga. 
Pernyataan tersebut merupakan tahapan perkembangan moral
a. Prakonvensional
b. Konvensional
c. Pascakonvensional
d. Interkonvensional 

SOAL KESULITAN BELAJAR SISWA 

10. Memaksimalkan kegiatan ekstrakurikuler, melakukan rekreasi dengan guru, dan


melakukan kegiatan informal lainnya memiliki fungsi untuk mengatasi kesulitan belajar
dalam hal ….
A. mengemukakan gagasan 
B. mengaktualisasikan diri
C. penciptaan hubungan yang baik 
D. menformulasikan tindakan 

11. Tindakan yang paling tepat dilakukan pada saat siswa mengalami kesulitan memahami
pelajaran yang sedang diajarkan adalah: 
A. Mengulang kembali bahan yang diajarkan
B. Memberikan tugas agar siswa mempelajari bahan yang belum dipahami 
C. Memberikan buku sumber untuk dipelajari siswa 
D. Membantu setiap siswa yang mengalami kesulitan 

12. Seorang peserta didik mampu mendengarkan informasi yang disampaikan oleh guru
tetapi pada saat ditanya ia tidak mengerti apa yang ia dengar. Peserta didik tersebut
mengalami kesulitan/ gangguan belajar dalam hal ....
A.      kesulitan akademis 
B.       gangguan simbolik
C.       gangguan nonsimbolik
D.      gangguan sosial 

13. Perbedaan antara konseling dan wawancara terletak pada maksud dan tujuannya. Tujuan
konseling adalah....
A.      Membantu siswa agar dapat memecahkan masalah pribadinya
B.       Menbantu siswa agar dapat melakukan penyesuaian diri dengan lingkungannya
C.      Membantu siswa agar dapat mengatasi kesulitan belajar
D.      Membantu siswa agar memperoleh informasi tertentu 

14. Cara yang yang paling cepat dan akurat yang digunakan untuk mengidentifikasi kesulitan
belajar peserta didik yang bersumber dari faktor sosial adalah …
A.      sosiometri
B.       angket
C.       wawancara
D.      brainstorming 
15. Bila anda sebagai guru menemukan peserta didik mengalami kesulitan memahami materi
pelajaran yang diberikan, tindakan apa yang akan dilakukan ?
A.      Memindahkantempat duduk peserta didik ke meja yang paling depan
B.       Mengulangi penjelasan bahan ajar kepada seluruh peserta didik
C.      Memberikan bantuan belajar kepada peserta didik yang bersangkutan 
D.      Menugaskan seluruh peserta didik membaca buku sumber 

16. Peserta didik banyak meluangkan waktu untuk bermain dengan teman-temannya. Dia rela
menghabiskan waktunya untuk teman daripada belajar. Ketika hasil tes dibagikan, serta
didik mendapat nilai yang kurang memuaskan. berusaha menyadarkan siswa. hal tersebut
merupakan penyadaran kesulitan belajar yang bersumber dari faktor ...
A.      keluarga
B.       sosial
C.       kondisi fisik 
D.      intelektual 

SOAL TEORI BELAJAR  

17. Teori ini memandang belajar sebagai hasil dari pembentukan hubungan antara
rangsangan dari luar (stimulus) dan balasan dari siswa (response) yang dapat diamati.
Semakin sering hubungan (bond) antara rangsangan dan balasan terjadi, maka akan
semakin kuatlah hubungan keduanya (law of exercise). Teori belajar yang dimaksud
adalah… 
A.      Behaviorisme 
B.       Humanistik 
C.       Sibernetik 
D.      Kontruktivisme 

18. Di dalam proses pembelajaran, para siswa dihadapkan dengan situasi di mana ia bebas
untuk mengumpulkan data, membuat dugaan (hipotesis), mencoba-coba (trial and error),
mencari dan menemukan keteraturan (pola), menggeneralisasi atau menyusun rumus
beserta bentuk umum, membuktikan benar tidaknya dugaannya itu. Hal ini merupakan
penerapan teori belajar….
A.      Sibernetik 
B.       Humannistik 
C.       Behaviorisme 
D.      Konstruktivisme 

19. Menurut teori ini, peranan guru dalam pembelajaran adalah sebagai fasilitator, motivator,
dan memberikan kesadaran mengenai makna kehidupan pada siswa. Teori belajar ini
berusaha memahami perilaku belajar dari sudut pandang pelakunya bukan dari sudut
pandang pengamatnya. Teori belajar ini adalah …. 
A.      Humanistik 
B.       Konstruktivisme 
C.       kognitivisme 
D.      Nativisme 

20. Pada masa kini siswa dituntut untuk dapat belajar setiap saat dan bisa terjadi di manapun.
Hal ini terjadi karena kemajuan teknologi yang memungkinkan belajar jarak jauh dalam
jaringan atau online. Pernyataan diatas sejalan dengan teori belajar ….
A.      Sibernetik 
B.       Konstruktivisme 
C.       Behaviorisme 
D.      Kognitivisme  

21. Pendapat yang menyatakan bahwa pengetahuan atau pengalaman yang baru dapat terkait
dengan pengetahuan lama yang sudah ada di dalam struktur kognitif seseorang adalah
teori belajar…
A.      Behaviorisme
B.       Konstruktivisme
C.      Kognitivisme
D.      Sibernatik 
SOAL PERENCANAAN DAN PELAKSANAAN PEMBELAJARAN  

22. Tujuan pembelajaran yang menggambarkan proses dan hasil belajar yang diharapkan
dicapai oleh peserta didik sesuai dengan kompetensinya dimuat dalam:
A.      Silabus
B.       RPP
C.       Silabus dan RPP
D.      SKL 

23. Komponen rancangan pelaksanaan pembelajaran terdiri dari....


A.    Identitas, kompetensi Inti, kompetensi dasar, indikator, tujuan pembelajaran,
materi, metode, media pembelajaran, sumber belajar, langkah-langkah
pembelajaran, evaluasi
B.   Identitas, kompetensi Inti, kompetensi dasar, tujuan pembelajaran, indikator,materi,
metode, media pembelajaran, sumber belajar, langkah-langkah pembelajaran, evaluasi
C.   identitas, kompetensi Inti, kompetensi dasar, indikator, tujuan pembelajaran, materi,
metode, sumber belajar, media pembelajaran, langkah-langkah pembelajaran, evaluasi
D.   Identitas, kompetensi Inti, kompetensi dasar, indikator, tujuan pembelajaran, materi,
metode, media pembelajaran, sumber belajar, evaluasi, langkah-langkah pembelajaran 

24. Silabus dan RPP sama-sama sebagai rencana proses pembelajaran, perbedaannya adalah
sebagai berikut:
A.   Silabus berisi kompetensi dasar sedangkan rpp mengarahkan kegiatan belajar
untuk mencapai kompetensi dasar
B.    Silabus bersumber dari standar isi dan standar lulusan, sedangkan RPP bersumber dari
standar kompetensi lulusan
C.   RPP dibuat oleh setiap guru, sedangkan silabus dibuat oleh tim guru
D.   RPP dan silabus keduanya disusun oleh setiap satan pendidikan. 

25. Salah satu prinsip dalam penyusunan rencana pelaksanaan pembelajaran (RPP) adalah 
A.      Materi atau bahan ajar berbasis kompetensi
B.       Pembelajaran dirancang dengan berpusat pada peserta didik
C.       RPP bersumber dari silabus 
D.      Alokasi waktu sesuai dengan jadwal pada setiap satuan pendidikan

26. Rencana pelaksanaan pembelajaran (RPP) disusun untuk setiap KD yang dapat
dilaksanakan dalam satu kali pertemuan atau lebih. Dijabarkan dari....
A.   Silabus untuk pembuatan indicator dan tujuan pembelajaran dalam upaya mencapai
KD
B.   Silabus untuk pembuatan tujuan pembelajaran dan materi dalam upaya mencapai KD
C.   Silabus untuk pembuatan ndikator dan penilaian dalam upaya mencapai KD
D.   Silabus untuk mengarahkan kegiatan belajar peserta didik dalam upaya
mencapai KD

27. Perhatikan beberapa komponen dari RPP untuk mata pelajaran bahasa Indonesia adalah
sebagai berikut ......
1)        Kompetensi dasar :Mengenal kegiatan bermusyawarah
2)        Indikator : menyebutkan dua ciri kegiatan bermusyawarah
3)        Tujuan Pembelajaran : setelah pelaksanaan pemilihan ketua kelas siswa dapat
menyebutkan dua ciri kegiatan musyawarah dengan benar
Berdasarkan komoponen-komponen RPP tersebut prinsip digunakan adalah....
A.      Memberikan umpan balik dan tindak lanjut
B.       Mendorong partisipasi aktif peserta didik
C.       Memperhatikan perbedaan individu peserta didik
D.      Keterkaitan dan keterpaduan

28. Dalam mengunakan media dan alat bantu mengajar, guru sebaiknya memperhatikan
beberapa faktor di bawah ini, kecuali satu yakni;:
A.      Metode mengajar yang digunakan guru
B.       Karakteristik bahan ajar 
C.       Waktu yang tersedia untuk kegiatan pembelajaran
D.      Buku sumber yang digunakan dalam pembelajaran
29. Jika seorang guru melakukan aktifitas pembelajaran sebagai berikut:
1)    Membekali murid tidak hanya dengan fakta-fakta, melainkan diarahkan pada
kemampuan penguasaan dalam proses berfikir dan berkomunikasi, 
2)    Berperan sebagai fasilitator dan pembimbing belajar peserta didik. 
3)    Melakukan penilaian yang mencakup cara-cara penyelesaian masalah dengan
berpatokan pada aturan yang berlaku, seperti peta konsep, diagram ven, portopolio,
uji kompetensi, dan ujian komprehensif.
Maka guru tersebut dapat dikatakan menggunakan pembelajaran yang berbasis....
A.      Kognitivisme
B.       Behaviorisme
C.       Konstructivisme
D.      Humanisme 

30. Menurut Piaget, periode perkembangan operasi kongkrit pada siswa usia peserta didik
antara 7 – 12 tahun. Dengan demikian proses pembelajaran dikelas harus lebih
menekankan…
A.      Penggunaan media, alat peraga termasuk sumber belajar.
B.       Pemberian tugas yang tersetruktur.
C.       Penggunaan metode bervariasi
D.      Menekankan berfikir ktiris dan pemecahan masalah.

31. Benjamin S. Bloom mengembangkan ranah kognitif dengan urutan berikut:


A.      ingatan; pemahaman, analisis, aplikasi, sintesis, dan evaluasi;
B.       ingatan, pemahaman, aplikasi, analisis, síntesis, dan evaluasi 
C.       ingatan, pemahaman, síntesis, aplikasi, analsis, dan evaluasi
D.      ingatan, pemahaman, analisis, aplikasi, evaluasi, dan, síntesis.

32. Taksonomi Anderson dan Krathwol menyempurnakan taksonomi Benjamin S. Bloom


mengembangkan ranah kognitif dengan urutan berikut:
A.      ingatan; pemahaman, analisis, aplikasi, sintesis, dan evaluasi;
B.       ingatan, pemahaman, aplikasi, analisis, síntesis
C.       ingatan, pemahaman, síntesis, aplikasi, analsis, dan evaluasi
D.      ingatan, pemahaman, aplikasi,analisis, evaluasi, dan, mencipta.

33. Model pembelajaran yang mempunyai keunggulan antara lain; berpikir dan bertindak
kreatif, memecahkan masalah yang dihadapi secara realistis, merangsang perkembangan
kemajuan berfikir siswa untuk menyelesaikan masalah yang dihadapi dengan tepat,
adalah….
A.      Role Playing
B.       Inquiry
C.       Problem Solving 
D.      Picture and Picture

34. Faktor yang penting dipertimbangkan guru dalam melaksanakan diskusi pemecahan
masalah proses pembelajaran adalah:
A.      Waktu yang tersedia untuk melaksanakan diskusi
B.       Rumusan masalah yang harus didiskusikan
C.       Jumlah peserta didik yang mengikti pembelajaran
D.      Motivasi belajar siswa

35. Kemampuan yang paling penting dimiliki guru tentang media dan alat bantu
pembelajaran adalah:
A.      Memilih bahan untuk membuat media/alat bantu
B.       Membuat media dan alat bantu sederhana
C.       Menggunakan media dan alat bantu 
D.      Merawat media dan alat bantu

36. Fungsi indikator dijadikan sebagai penanda dalam….    


A.      Pencapaian standar kompetensi
B.       Pencapaian kompetensi dasar. 
C.       Pencapaian tujuan pembelajaran
D.      Pencapaian standar kelulusan
37. Tindakan yang paling tepat dilakukan pada saat siswa mengalami kesulitan memahami
pelajaran yang sedang diajarkan adalah: 
A.      Mengulang kembali bahan yang diajarkan
B.       Memberikan tugas agar siswa mempelajari bahan yang belum dipahami 
C.       Memberikan buku sumber untuk dipelajari siswa 
D.      Membantu setiap siswa yang mengalami kesulitan 

38. Bila anda sebagai guru menemukan peserta didik mengalami kesulitan memahami materi
pelajaran yang diberikan, tindakan apa yang akan dilakukan ?
A.      Memindahkantempat duduk peserta didik ke meja yang paling depan
B.       Mengulangi penjelasan bahan ajar kepada seluruh peserta didik
C.       Memberikan bantuan belajar kepada peserta didik yang bersangkutan 
D.      Menugaskan seluruh peserta didik membaca buku sumber

39. Apa yang paling tepat dilakukan guru, jika seorang peserta didik tidak memberikan
jawaban atas pertanyaan yang diajukan ?
A.      Menunjuk peserta didik lain untuk menjawab pertanyaan tersebut
B.       Memberikan kritik agar peserta didik berusaha menjawab walaupun salah
C.       Menyederhanakan isi pertanyaan agar mudah dipahami peserta didik 
D.      Menjawab sendiri pertanyaan tersebut.

40. Prinsip sistematis sebagai salah satu prinsip pengembangan silabus artinya ....
A.Cakupan, kedalaman, tingkat kesukaran dan urutan penyajian materi dalam silabus
sesuai dengan tingkat perkembangan fisik, intelektual, sosial, emosional, dan
spritual peserta didik. 
B. Adanya hubungan yang konsisten (ajeg, taat asas) antara kompetensi dasar,
indikator, materi pokok/pembelajaran, pengalaman belajar, sumber belajar, dan
sistem penilaian.
C.Cakupan indikator, materi pokok/pembelajaran, pengalaman belajar,
sumber belajar, dan sistem penilaian cukup untuk menunjang pencapaian
kompetensi dasar.
D. Komponen-komponen silabus saling berhubungan secara fungsional dalam
mencapai kompetensi.

41. Ciri-ciri kelas yang menggunakan pembelajaran CTL adalah sebagai berikut, kecuali....
A.      Sharing dengan teman
B.       Guru kreatif
C.       Pengalaman nyata
D.      Menggunakan satu sumber

42. Dalam memilih dan mengembangkan materi pembelajaran, Ibu Murni Nugroho selalu
menyeleksi materi pembelajaran yang telah teruji kebenarannya, tidak ketinggalan jaman
dan memberikan kontribusi untuk pemahaman ke depan. Kriteria penyeleksian dan
pemilihan materi pembelajaran tersebut memenuhi kriteria…
A.      Sahih
B.       Kebermanfaatan
C.      Layak dipelajari
D.      Menarik minat

43. Bu Rossa menyiapkan materi pelajaran dengan cara dimulai dari materi-materi yang
dikenal siswa kemudian menuju hal-hal baru dan dianggap lebih mendalam. Bentuk
pengembangan materi yang disusun Bu Rossa mengikuti pola....
A.      Kausal
B.       Spiral
C.      Kronologis
D.      inquiri
SOAL MEDIA DAN TIK DALAM PEMBELAJARAN 
44. Memilih media pembelajaran hendaknya tidak boleh sembarangan tetapi harus
didasarkan pada kriteria tertentu. Misalnya, apakah untuk belajar individual, kelompok
kecil, kelompok besar atau massal. 
Pernyataan tersebut dalam pemilihan media termasuk ke dalam kriteria...         
A.      tujuan            
B.       sasaran didik 
C.       ketersediaan  
D.      konteks penggunaan

45. Pada saat mempersiapkan pembelajaran seorang guru dapat menyusun strategi
pembelajaran dan menentukan media yang akan digunakan dalam pembelajaran tersebut.
Kemampuan dasar yang harus dimiliki seorang pendidik terkait dengan keterampilan
memilih media pembelajaran adalah…
A.      Pendidik harus mengetahui latar sosial budaya siswa dan sekolah
B.       Pendidik harus memahami karakteristik dari media pembelajaran tersebut.
C.       Pendidik harus menyesuaikan diri dengan kemampuan sekolah.
D.      Pendidik menyesuaikan dengan materi pembelajaran.

46. Pada saat mempersiapkan pembelajaran seorang guru harus dapat menentukan jenis
media yang tepat sesuai dengan materi. Di lihat dari kemampuan jangkauannya, media
dapat dibagi ke dalam.. 
A.   media yang memiliki daya liput yang luas dan media yang memiliki daya liput
terbatas        
B.    media yang dpat didengar saja dan media yang dapat dilihat         
C.    media tradisional dan media berbasis TIK   
D.   media dua dimensi dan media tiga dimensi 

47. Di ruang media tersedia berbagai macam media yang sudah berdebu dan agak usang.
Sebagai seorang pendidik yang kreatif, sebaiknya dapat memanfaatkan media dan
memilih media mana yang akan digunakan. Dia memiliki beberapa pertimbangan dalam
hal ini. 
Pertimbangan dalam memilih media pembelajaran yang harus diperhatikan adalah...
A.   Tujuan, sasaran didik, karakteristik media, waktu pengoperasian, biaya,
ketersediaan,konteks penggunaan, dan mutu teknis.
B.   Sasaran didik, karakteristik media, waktu mengoperasikan, tujuan, misi visi
sekolah dan konteks penggunaan.
C.   Tujuan, sasaran didik, karakteristik media, visi sekolah dan konteks penggunaan 
D.   Kultur sekolah, tujuan, waktu pengoperasiaan dan karakteristik media

48. Seorang guru harus mampu memanfaatkan media pembelajaran dan sumber belajar untuk
mencapai tujuan pembelajaraan utuh. Pernyataan berikut yang benar terkait dengan
media pembelajaran adalah:          
A.  media pembelajaran yang paling baik adalah media yang berbasis TIK      
B.  sebuah media dapat digunakan untuk semua kegiatan pembelajaran           
C.  media dapat digunakan sebagai pembawa pesan dalam suatu kegiatan
pembelajaran          
D.   memilih media tidak perlu banyak pertimbangan agar tidak merepotkan

49. Setiap materi pembelajaran memiliki tingkat kesukaran yang bervariasi. Untuk
memudahkan peserta didik memahami materi yang memiliki tingkat kesukaran tinggi
guru sering memanfaatkan media pembelajaran. Misalnya, media gambar atau tayangan
video yang berisi sistem peredaran darah. 
Fungsi media pada pernyataan tersebut adalah:  
A.      Menampilkan objek yang terlalu besar         
B.       Membuat konkrIt konsep yang abstrak   
C.       Menampilkan objek yang tidak dapat diamati dengan mata telanjang.       
D.      Membawa objek yang berbahaya atau sukar didapat di dalam lingkungan belajar.
50. Media memiliki fungsi dan manfaat bagi pembelajaran. Hal ini dirasakan juga oleh guru
dalam membantu pembelajaran menjadi lebih efisien dan efektif. Misalnya saat guru
ingin menjelaskan suatu konsep/ objek yang luas dan besar, dia tinggal menggunakan
medianya saja. 
Berikut ini adalah salah satu fungsi dari media pembelajaran sesuai kondisi tersebut:
A.  Menampilkan objek yang terlalu besar, misalnya pasar, candi.
B.  Membawa objek yang berbahaya atau sukar didapat di dalam lingkungan belajar.
C.  Membuat konkret konsep yang abstrak, misalnya untuk menjelaskan peredaran darah.
D.  Menampilkan objek yang tidak dapat diamati dengan mata telanjang.

51. Seorang guru ingin membuka situs google untuk menuliskan catatan, ide, atau refleksi
yang bersifat pribadi atau untuk dibagikan secara umum. Fitur yang dapat dimanfaatkan
oleh guru tersebut adalah…         
A. book  
B. forum            
C  blog   
D. e-portofolio

52. Seorang guru sedang ber-googling atau membuka situs google untuk mencari informasi
mengenai pembelajaran berbasis TIK (Teknologi Informasi dan Komunikasi). 
Pengertian googling dalam istilah internet tersebut adalah… 
A.  program mencari informasi   
B.  program desktop publishing 
C.  program untuk menjelajahi laman 
D. program penciptaan laman

53. Seorang guru yang akan mencari informasi tentang suatu topik atau judul dapat
menggunakan aplikasi internet yang dikenal dengan istilah mesin pencari (search engine).
Salah satu laman yang berfungsi sebagai mesin pencari adalah... 
A.      www.gmail.com       
B.       www.google.com 
C.       www.hotmail.com 
D.      www.cari-data.com

54. Saat kita akan informasi tentang suatu topik atau judul dapat menggunakan aplikasi
internet yang dikenal dengan istilah mesin pencari (search engine). 
Salah satu laman yang berfungsi sebagai mesin pencari selain google adalah... 
A.  www.yahoo.com
B.   www.gmail.com 
C.  www.hotmail.com 
D.  www.cari-data.com

55. Seorang guru akan membuat media untuk menampilkan contoh surat, tabel, gambar dan
berbagai dokumen lain. Aplikasi sederhana keluaran Microsoft yang dapat digunakan
untuk memfasilitasi kegiatan guru tersebut adalah...  
A. Microsoft Database     
B. Microsoft PowerPoint             
C. Microsoft Desktop Publishing            
D. Microsoft Word 

56. Seorang guru dapat menampilkan bahan tayang yang menarik perhatihan siswa dalam
pembelajaran. Program tampilan bahan tayang ini difasilitasi oleh Microsoft. Aplikasi
sederhana keluaran Microsoft yang dapat digunakan untuk membuat bahan presentasi
adalah.... 
A.  Microsoft Database 
B.  Microsoft Spread Sheet 
C.  Microsoft powerpoint 
D.   Microsoft Desktop Publishing
57. Tools pembelajaran daring/ online cukup banyak dan memiliki fungsi masing-masing.
Salah satu fiturnya berupa kumpulan soal tes sumatif dan tes akhir pembelajaran.
Fitur moodle ini adalah... 
A.      Quiz 
B.       Page 
C.       Lesson           
D.      Blog

58. Dalam pembelajaran daring terdapat beberapa unsur atau elemen pembangun
pembelajaran elektronik. Unsur berikut yang bukan termasuk unsur pembangun
elektronik adalah…      
A.      Penulis konten (content writer)        
B.       Penata grafis (design grafis)          
C.       perancang pembelajaran (course design)      
D.      pemrogram komputer (computer programmer).

59. Dalam mengawali pelajarannya, Pak Ali merumuskan pertanyaan, mengumpulkan data
(informasi) dengan berbagai teknik, mengasosiasi/ menganalisis/mengolah data (informasi)
dan menarik kesimpulan serta mengkomunikasikan hasil yang terdiri dari kesimpulan untuk
memperoleh pengetahuan, keterampilan dan sikap.
Langkah yang dilakukan Pak Ali itu merupakan bagian dari model pembelajaran…
A.      portofolio
B.       saintifik
C.       penemuan
D.      autentik

60. Dalam mengawali pelajarannya, Pak Mamaz Nendar menyampaikan suatu permasalahan
guna menggugah dan menimbulkan kepenasaran-kepenasaran para siswanya. Langkah
tersebut bertujuan untuk mendorong para siswa agar mau mengembangkan kreativitasnya
Langkah yang dilakukan Pak Mamaz Nendar itu merupakan bagian dari model
pembelajaran….
A.      portofolio
B.       saintifik
C.      penemuan
D.      autentik

61. Pak Larso melaksanakan pembelajaran dengan menggunakan masalah sebagai langkah
awal dalam mengumpulkan dan mengintergrasikan pengetahuan baru berdasarkan
pengalamannya, dimulai dengan memunculkan pertanyaan penuntun (a guiding question)
dan membimbing peserta didik berkolaboratif yang mengintegrasikan berbagai subjek
(materi) dalam kurikulum. 
Pembelajaran yang dilakanakan oleh Pak Larso tersebut merupakan model
pembelajaran.... 
A.      projek based learning 
B.       discovery Learning
C.       Problem Based Learning
D.      Inquiry learning

62. Bu Rumini melaksanakan pembelajaran yang mengakomodasi semua anggota kelompok


mengungkapkan pendapat, ide, dan tanggapan terhadap skenario secara bebas, sehingga
dimungkinkan muncul berbagai macam alternatif pendapat. 
Kegiatan yang dilakukan Bu Rumini tersebut merupakan implementasi model
pembelajaran .
A.      Project based learning
B.       inquiry learning
C.       Discovery learning 
D.      problem based learning
63. Ketika memulai pembelajaran, Bu Dwi Wahyu menyodorkan sebuah isu aktual dalam
bentuk pertanyaan yang memerlukan solusi. Lalu, dalam bentuk kelompok-kelompok
kecil, siswa diminta untuk untuk mencari alternatif solusi tersebut. 
Kegiatan yang dilakukan Bu Dwi Wahyu tersebut mengisyaratkan implementasi model
pembelajaran ....
A.      Project based learning
B.       Problem based learning
C.       Discovery learning
D.      Inquiry learning

64. Sebelum memulai proses belajar-mengajar di dalam kelas, Bu Nuraini meminta peserta
didik terlebih dahulu untuk mengobservasi suatu fenomena. Kemudian peserta didik
diminta mencatat masalah-masalah yang muncul. Setelah itu, peserta didik dirangsang
untuk berpikir kritis dalam memecahkan masalah yang ada. Sementara itu, Bu Nuraeni
mengarahkan peserta didik untuk bertanya, membuktikan asumsi, dan mendengarkan
pendapat yang berbeda dari mereka. 
Kegiatan yang dilakukan Bu Nuraeni tersebut mengisyaratkan implementasi model
pembelajaran 
A.      inquiry learning
B.       problem based learning
C.       Project based learning
D.      Discovery learning

65. Pak Kandi bersama siswanya merencanakan sebuah kegiatan secara kolaboratif. Hal ini
dimaksudkan agar peserta didik merasa “memiliki” dan bertanggung jawab atas kegiatan
yang telah direncanakan bersama itu. Dalam perencanaan itu dibicarakan mengenai
aturan main, aktivitas pendukung, alat dan bahan yang dapat diakses, cara dan langkah
kerja untuk menyelesaian rencana kegiatan kolaboratif tadi.
Model pengelolaan pembelajaran yang dipraktikkan oleh Pak Kandi mencerminkan
prinsip model ....
A.      Problem solving learning
B.       Problem based learning
C.      Project based learning
D.      Discovery learning

66. Berikut ini merupakan langkah-langkah pembelajaran dengan model pembelajaran


project based learning : 
1) Penentuan Pertanyaan Mendasar, 
2) Mendesain Perencanaan Proyek, 
3) Menyusun Jadwal (Create a Schedule), 
4) Memonitor peserta didik dan kemajuan proyek (Monitor the Students and the
Progress of the Project), 
5) Menguji Hasil (Assess the Outcome), 
6) Mengevaluasi Pengalaman (Evaluate the Experience). 
Urutan yang benar adalah .... 
A.      1-2-3-4-5-6
B.       1-3-2-4-5-6
C.       1-3-2-5-6-4
D.      1-2-3-5-4-6

67. Perhatikan langkah-langkah kegiatan pembelajaran yang masih acak berikut ini! 
1)        Mengorganisasi siswa dalam belajar 
2)        Orientasi siswa pada masalah 
3)        Membimbing penyelidikan siswa secara mandiri atau kelompok 
4)        Mengembangkan dan menyajikan hasil karya 
Jika langkah-langkah tersebut disusun mengikuti urutan model pembelajaran Problem
Based Learning, urutan langkah yang tepat adalah ....
A.      1-2-3-4
B.       2-3-1-4
C.       3-2-1-4
D.      2-3-1-4
68. Perhatikan langkah-langkah kegiatan pembelajaran berikut ini.
1)        Orientasi siswa pada masalah 
2)        Mengorganisasi siswa dalam belajar 
3)        Membimbing penyelidikan siswa secara mandiri atau kelompok 
4)        Mengembangkan dan menyajikan hasil karya 
5)        Menganalisis dan mengevaluasi proses pemecahan masalah
Fase-fase pembelajaran tersebut merupakan fase dalam pembelajaran dengan model ....
A.      discovery Learning
B.       Project Based Learnin
C.       problem solving learning 
D.      problem based learning

SOAL PENILAIAN 
69. Kriteria keberhasilan belajar siswa ditentukan dengan menggunakan Kriteria Ketuntasan
Minimal (KKM). KKM adalah rata-rata setiap unsur dari kriteria yang ditentukan. Untuk
menentukan KKM diperlukan faktor-faktor….  
A.      Kompleksitas indikator, daya dukung, dan kemampuan guru
B.       Kemampuan guru, sarana/prasarana, dan intake siswa
C.       Kompleksitas indikator, daya dukung, dan intake siswa
D.      Kemampuan guru, tingkat kesulitan kompetensi dasar, dan intake siswa

70. Fungsi KKM adalah sebagai berikut, kecuali....


A.   sebagai acuan peserta didik dalam menyiapkan diri mengikuti penilaian mata
pelajaran
B.   merupakan target satuan pendidikan dalam pencapaian kompetensi tiap mata
pelajaran
C.   dapat digunakan sebagai bagian dari komponen dalam melakukan evaluasi program
pembelajaran yang dilaksanakan di sekolah.
D.  Sebagai kegiatan pengambilan keputusan yang dapat dilakukan melalui metode
kualitatif atau kuantitatif. 

71. Fungsi indikator dijadikan sebagai penanda dalam….    


A.      Pencapaian standar kompetensi
B.       Pencapaian kompetensi dasar. 
C.       Pencapaian tujuan pembelajaran
D.      Pencapaian standar kelulusan

72. Untuk menentukan tindak lanjut pembelajaran, tindakan yang perlu dilakukan guru
adalah;
A.      Menilai tingkat pencapaian hasil belajar siswa
B.       Menambah bahan pelajaran baru 
C.       Mengetahui jumlah siswa yang tidak akan mengikuti program tindak lanjut
D.      Memilih waktu yang tepat untuk melaksanakan program tindak lanjut

73. Instrumen yang diguanakan untuk menilai perilaku siswa dalam proses pembelajaran
adalah:
A.      Pedoman observasi
B.       Kuesioner
C.       Pedoman wawancara
D.      Tes hasil belajar

74. Bila Anda sebagai guru menemukan peserta didik mendapatkan hasil belajar yang
rendah, tindakan apa yang akan dilakukan ?
A.      Memindahkan tempat duduk peserta didik ke meja yang paling depan
B.       Mengulangi penjelasan bahan ajar kepada seluruh peserta didik
C.       Memberikan bantuan belajar kepada peserta didik yang bersangkutan 
D.      Menugaskan seluruh peserta didik membaca buku sumber
75. Sebelum guru menyusun soal-soal untuk menilai hasil belajar peserta didik, manakah
yang pertama kali harus dipelajari:
A.      Buku sumber yang digunakan
B.       Kurikulum dan silabus
C.       Indikator pencapaian kompetensi
D.      Kemampuan awal siswa

76. Salah satu prinsip dalam penilaian hasil belajar peserta didik adalah penilaian harus
terpadu artinya:
A.   penilaian berdasarkan data yang mencerminkan kemampuan yang harus diukur
B.   penilaian merupakan bagian yang tak terpisahkan dari kegiatan pembelajaran
C.   penilian berdasarkan kriteria yang jelas tentang kompetensi yang harus dicapai
D.   penilaian harus berkesinambungan oleh pendidik untuk semua aspek
kompetensi

77. Instrumen penilaian hasil belajar yang digunakan pendidik harus memenuhi persayaratan
berikut, kecuali:
A.  Konstruksi butir soal memenuhi persayaratan yang sesuai dengan bentuk tes yang
digunakan
B.  Substansi yang diukur mempresentasiukan kompetensi yang dinilai
C.  Bahasa yang digunakan komukinatif sesuai dengan taraf perkembangan peserta didik
D.  Digunakan untuk ulangan tengah semester dan ulangan akhir semester

78. Kriteria ketuntasan minimal (KKM) merupakan kriteria ketuntasan belajar yang
ditentukan oleh sekolah, pada prinsipnya merupakan:
A.      Nilai batas ambang kompetensi
B.       Nilai minimal yang dicapai peserta didik
C.       Nilai maksimal yang dicapai peserta didik
D.      Nilai batas ambang kelulusan ujian sekolah

79. Untuk mengetahui prestasi belajar yang dicapai oleh semua peserta didik pada satu
rombongan belajar dapat dilakukan dengan menghitung nilai;
A.      Mean
B.       Modus
C.       Median
D.      Simpangan baku

80. Standar penilaian pendidikan merupakan acuan bagi guru dalam melaksanakan
A.      Penilaian hasil belajar peserta didik
B.       Penilaian proses pembelajaran yang dilakukan guru
C.       Penilaian silabus dan RPP
D.      Penilaian standar kompetensi lulusan

81. Penilaian adalah penafsiran hasil pengukuran dan penentuan pencapaian hasil belajar.
Penilaian dapat berupa hasil belajar peserta didik yang tidak dipengaruhi oleh
kepentingan penilai, perbedaan latar belakang agama, sosial-ekonomi, budaya, bahasa,
gender, dan hubungan emosional. Cara penilaian tersebut mengacu pada aspek
penilaian….            
A.      Objektivitas.
B.       Transparan
C.       Bermakna
D.      Menyeluruh

82. Ketika Anda tahu bahwa guru telah berhasil atau tidak dalam kegiatan pembelajaran, apa
yang Anda lakukan?
A.      Membuat laporan keberhasilan/ketidakberhasilan
B.       Menyampaikan laporan ke atasannya
C.       Menyusun rencana tindak lanjut
D.      Menyusun laporan hasil pemantauan
83. Upaya merancang pengayaan bagi perserta didik yang mencapai ketuntasan belajar
optimal tampak dalam kegiatan guru sebagai berikut:
A. Memberikan tambahan materi berupa sumber ajar dari pengarang yang berbeda
B. memberikan test tambahan dengan tingkat kesukaran lebih tinggi
C. memberikan tambahan sumber bacaan yang lebih mendalam dan tingkat variasi yang
tinggi berikut instrument testnya yang sesuai
D.  diberikan materi bahan ajar yang lebih tinggi tingkatannya dan mengerjakan
soal-soal yang memiliki kesulitan tinggi 

84. Dasar rancangan program remidi bagi peserta didik yang capaian prestasinya di bawah
ketuntasan belajar ….
A.      proses pengajaran remedial pada dasarnya adalah proses belajar mengajar biasa
B.       tujuan pengajaran remedial adalah sama dengan test diagnostik
C.       sasaran terpenting pengajaran remidial adalah peningkatan kecerdasan siswa
D.      strategi yang dipilihhanya berbentuk test ulang

85. Salah satu prinsip merancang program remidial bagi peserta didik tampak dalam kegiatan
guru ….
A.  Membuat rancangan pembelajaran khusus untuk siswa peserta remedial
B.  Menggunakan rancangan pembelajaran yang telah dibuat dengan memperhatikan
hasil temuan analisis evaluasi belajar siswa
C.  Menggunakan rancangan pembelajaran baru yang berbeda sama sekali dengan
rancangan yang ada.
D.   merancang test ulang saja tanpa ada pengulangan penjelasan materi

86. Upaya guru menggunakan hasil analisis untuk menentukan ketuntasan belajar antara lain
sebagai berikut....
A. Menentukan kriteria keberhasilan belajar
B. Mengklasifikasi siswaberdasarkan hasil capaian belajarnya
C.  Mencari letak kelemahan secara umum dilihat dari kriteria keberhasilan yang
diharapkan
D.  Merencanakan pengajaran remid

87. Penilaian portofolio dapat dilaksanakan dengan cara ….


A.  Memberikan penilaian menyeluruht erhadap tugas-tugas siswa
B.  Mengumpulkan lembaran-lembaran jawaban hasil test harian dan sumatif tiap siswa
C.   Mengumpulkan hasil kerja masing-masing siswa yang telah diberikan masukan
baik oleh guru dan rekan siswa dalam suatu album sebagai bukti hasil belajar
D.   Mengumpulkan lembaran-lembaran jawaban hasil ulangan tiap siswa untuk melihat
kesulitan siswa dalam memahami pokok bahasan tertentu dan kemudian diberikan
pengajarandan test remedial

88. Penilaian hasil belajar siswa didasarkan pada karya siswa dan tugas siswa, kemampuan
dalam proses pembelajaran dan hasil post test disebut ....
A.      Konstruktivisme
B.       Authentic assesment
C.       Efektif
D.      kondusif 

SOAL PENDEKATAN PENILAIAN

89. Pendekatan tes yang berusaha menafsirkan hasil tes yang diperoleh siswa dengan
membandingkannya dengan patokan telah ditetapkan, sebelum hasil tes itu sendiri
diperoleh, dan bahkan sebelum kegiatan pengajaran dilakukan, patokan yang akan
dipergunakan untuk menentukan batas kelulusan itu telah ditetapkan dinamakan .... 
A.      penilaian acuan patokan 
B.       penilaian acuan norma 
C.       penilaian tuntas 
D.      penilaian berkelanjutan
90. Pendekatan tes ini menggunakan norma yang disusun secara relatif berdasarkan distribusi
skor yang dicapai oleh para pengikut dalam suatu tes. Dengan demikian maka skor
standar yang dicapai oleh seseorang yang didasarkan atas norma relatif ini (PAN)
mencerminkan status individu di dalam kelompok. 
Pendekatan tes yang dimaksud adalah….
A.      Penilaian berkelanjutan
B.       penilaian menyeluruh
C.       penilaian acuan patokan
D.      penilaian acuan norma 

91. Kurikulum 2013 menggunakan pendekatan penilaian yang disebut dengan istilah
penilaian acuan kriteria (PAK). PAK merupakan penilaian pencapaian kompetensi yang
didasarkan pada kriteria ketuntasan minimal (KKM). KKM merupakan kriteria
ketuntasan belajar minimal yang ditentukan oleh satuan pendidikan dengan
mempertimbangkan karakteristik Kompetensi Dasar yang akan dicapai, daya dukung, dan
karakteristik peserta didik. 
Dengan demikian, Kurikulum 2013 menggunakan pendekatan penilaian yang didasarkan
pada pendekatan ....
a.         Penilaian berkelanjutan
b.        Penilaian autentik
c.         Penilaian acuan patokan
d.        Penilaian acuan norma

SOAL TUJUAN, FUNGSI, DAN PRINSIP PENILAIAN 

92. Menggambarkan sejauh mana seorang peserta didik telah menguasai suatu kompetensi,
merupakan… 
A.      tujuan penilaian 
B.       prinsip penilaian 
C.      fungsi penilaian 
D.      hasil penilaian

93. Mengetahui tingkat penguasaan kompetensi dalam sikap, pengetahuan, dan keterampilan
yang sudah dan belum dikuasai seorang/sekelompok peserta didik untuk ditingkatkan
dalam pembelajaran remidial dan program pengayaan merupakan…. 
A.      prinsip penilaian
B.       tujuan penilaian 
C.       fungsi penilaian 
D.      bentuk penilaian.
94. Fungsi penilaian yang tepat adalah …
A.  alat meningkatkan prestasi peserta didik 
B.  alat untuk menemukan gaya belajar peserta didik 
C. membantu peserta didik memahami materi pembelajaran
D. sebagai kontrol bagi pendidik dan satuan pendidikan tentang kemajuan
perkembangan peserta didik.

95. Assesment hasil belajar peserta didik harus memperhatikan prinsip-prinsip sebagai
berikut, kecuali: 
A.      objektif 
B.       adil 
C.      kooperatif 
D.      terpadu.

96. Penilaian yang didasarkan pada data yang mencerminkan kemampuan yang diukur, hal
tersebut merupakan prinsip penilian yang: 
A.      adil 
B.       objektif 
C.      valid 
D.      sistematis
97. Penilaian yang dapat dipertanggungjawabkan, baik dari segi teknik, prosedur, maupun
hasilnya, hal tersebut merupakan prinsip penilaian yang… 
A.      adil 
B.      akuntabel 
C.       valid 
D.      sistematis 

98. Penilaian yang dilakukan secara berencana dan bertahap dengan mengikuti langkah-
langkah yang baku, hal tersebut merupakan prinsip penilaian yang… 
A.      adil 
B.       objektif 
C.       valid 
D.      sistematis 

99. Di bawah ini beberapa hal yang perlu diperhatikan dalam melakukan assesmen hasil
belajar peserta didik, kecuali…. 
A.      ditujukan untuk mengukur pencapaian kompetensi
B.       menggunakan acuan kriteria berdasarkan pencapaian kompetensi
C.       ditindaklanjuti dengan program remedial dan pengayaan
D.      dilakukan pengulangan jika ternyata hasilnya banyak yang jelek

SOAL PROGRAM REMEDIAL DAN PENGAYAAN 

100. Program pembelajaran remedial hendaknya memungkinkan peserta didik untuk


belajar sesuai dengan kecepatan, kesempatan, dan gaya belajar masing-masing.
Pembelajaran remedial harus mengakomodasi perbedaan individual peserta
didik.Pernyataan di atas termasuk salah prinsip yang perlu diperhatikan dalam
pembelajaran remedial, yaitu... 
A.      Fleksibilitas
B.       Interaktif
C.      Adaptif
D.      Kesinambungan

101. Berdasarkan data hasil evaluasi pembelajaran tentang memahami teks anekdot
ternyata hasilnya tidak maksimal. Dari 30 siswa dinyatakan belum tuntas sejumlah 15
sehingga mengikuti program remidial. Sedangkan yang dinyatakan tuntas sejumlah 15
orang mengikuti program pengayaan. Kegiatan pengayaan untuk 15 siswa dapat
dilakukan oleh guru dengan cara... 
A.      Mengadakan pendalaman materi terkait dengan KD tersebut
B.       Digabung dengan siswa yang belum tuntas ikut remedial
C.       Melanjutkan materi pada KD selanjutnya 
D.      Memberi tugas mengerjakan Lembar Kerja Siswa 

102. Upaya merancang pengayaan bagi perserta didik yang mencapai ketuntasan
belajar optimal tampak dalam kegiatan guru sebagai berikut:
A.  Memberikan tambahan materi berupa sumber ajar dari pengarang yang berbeda
B.  memberikan test tambahan dengan tingkat kesukaran lebih tinggi
C.  memberikan tambahan sumber bacaan yang lebih mendalam dan tingkat variasi yang
tinggi berikut instrument testnya yang sesuai
D.  diberikan materi bahan ajar yang lebih tinggi tingkatannya dan mengerjakan
soal-soal yang memiliki kesulitan tinggi

103. Dasar rancangan program remidi bagi peserta didik yang capaian prestasinya di
bawah ketuntasan belajar ….
A.  proses pengajaran remedial pada dasarnya adalah proses belajar mengajar biasa
B.   tujuan pengajaran remedial adalah sama dengan test diagnostik
C.  sasaran terpenting pengajaran remidial adalah peningkatan kecerdasan siswa
D.  strategi yang dipilihhanya berbentuk test ulang
104. Salah satu prinsip merancang program remidial bagi peserta didik tampak dalam
kegiatan guru
A.    Membuat rancangan pembelajaran khusus untuk siswa peserta remedial
B.     Menggunakan rancangan pembelajaran yang telah dibuat dengan
memperhatikan hasil temuan analisis evaluasi belajar siswa
C.    Menggunakan rancangan pembelajaran baru yang berbeda sama sekali dengan
rancangan yang ada.
D.   merancang test ulang saja tanpa ada pengulangan penjelasan materi

SOAL PTK 
105. Agar dapat menyusun hipotesis tindakan dengan tepat, Anda dapat melakukan
kegiatan-kegiatan. Salah satu kegiatan itu adalah pengkajian teoretik di bidang
pembelajaran/pendidikan.Pernyataan tersebut merupakan kegiatan PTK pada langkah ...
A.      merumuskan masalah 
B.       mengidentifikasi masalah
C.      Merancang PTK dengan mengajukan hipotesis tindakan
D.      menyusun proposal penelitian

106. Mengetahui proses tindakan, pengaruh tindakan (yang disengaja dan tak sengaja),
(c) keadaan dan kendala tindakan, (d) bagaimana keadaan dan kendala tersebut
menghambat atau mempermudah tindakan dan pengaruhnya merupakan kegiatan PTK
pada langkah ….
A.      refleksi
B.       observasi
C.       perencanaan 
D.      pelaksanaan

107. Model rancangan PTK terletak pada alur pelaksanaan tindakan yang dilakukan.
Hal ini sekaligus menjadi penanda atau ciri khusus yang membedakan PTK dengan jenis
penelitian lain. Adapun alur penelitian tindakan yang dimaksud adalah ….
A.      A. observasi --> refleksi --> perencanaan --> pelaksanaan tindakan
B.       refleksi --> perencanaan --> pelaksanaan tindakan --> observasi 
C.       perencanaan --> observasi --> pelaksanaan tindakan --> refleksi 
D.      perencanaan --> pelaksanaan tindakan --> observasi --> refleksi 

108. Rumusan masalah dalam PTK berikut, yang mana paling tepat disebut sebagai
rumusan masalah PTK?
A.  Apakah pengaruh permainan peran dapat meningkatkan keaktifan siswa kelas XI
SMA Negeri 2 Boyolali?
B.   Bagaimana cara meningkatkan kemampuan siswa dalam mengajukan
pertanyaan melalui metode tanya jawab dalam pelajaran Bahasa Indonesia di
Kelas XI IPA 2?
C.  Mengapa siswa SMA Negeri 2 Boyolali selalu menjadi juara dalam berbagai
perlombaan nasional?
D.  Bagaimana cara menggunakan alat peraga yang berasal dari lingkungan sekitar
sekolah?

109. Dalam Penelitian tindakan kelas, masalah yang diteliti berasal dari ....
A.      kerisauan guru akan kinerjanya di kelas yang diajar
B.       keriasauan pendidik akan mutu pendidikan
C.       keinginan untuk membantu guru
D.      kepedulian peneliti akan kinerja guru

110. Di bawah ini adalah prinsip-prinsip penelitian tindakan kelas (PTK) menurut
Suharsimi Arikunto, kecuali….
A.      Kesadaran diri untuk memperbaiki kinerja
B.       Kegiatan yang direkayasa
C.       Perencanaan
D.      Upaya empiris dan sistematis
111. Pola perencanaan-pelaksanaan-observasi-refleksi secara bertahap dan terus-
menerus merupakan karakteristik PTK dalam hal….
A.      Pengumpulan data
B.       Menganalisis masalah
C.       Mencapai tujuan
D.      Menentukan ruang lingkup

112. Peran guru dalam PTK adalah sebagai…


A.      Guru
B.       Peneliti
C.      Guru dan Peneliti
D.      Objek Penelitian

113. Berikut ini kegiatan PTK pada tahap pertengahan tindakan, kecuali….
A.      Pelaksanaan tindakan
B.       Observasi dan intepretasi
C.       Diskusi balikan
D.      Analisis data

114. Tujuan PTK adalah…


A.      Perbaikan KBM
B.       Menyusun teori baru
C.       Memperbaiki teori
D.      Menyusun generalisasi

115. Merenungkan kembali tindakan perbaikan dan dampaknya serta mencari jalan
keluar untuk tindak lanjut PTK merupakan…
A.      Analisis
B.       Refleksi
C.       Observasi
D.      Diskusi

116. Latar belakang, tujuan penelitian, dan manfaat penelitian harus menjadi bagian
dari PTK khususnya pada bagian….
A.      Kesimpulan
B.       Prosedur penelitian
C.      Pendahuluan
D.      Metodologi penelitian

117. Persiapan awal dalam PTK adalah…


A.      Menyiapkan cara wawancara
B.       Membuat analisis kelayakan hipotesis
C.      Membuat rencana (perbaikan) pembelajaran dan skenarionya
D.      Menyiapkan fasilitas dan sarana

118. Proposal penelitian dalam PTK adalah….


A.      Usulan untuk mendapatkan dana penelitian
B.       Uraian tentang komponen-komponen yang harus dilakukan guru
C.       Uraian projek penelitian pendidikan
D.      Perencanaan sistematik untuk melaksanakan PTK

119. Berikut ini langkah-langkah dalam tahap perencanaan tindakan dalam PTK,
kecuali…
A.      Formulasi solusi dalam bentuk hipotesis tindakan
B.       Analisis kelaikan hipotesis tindakan
C.       Persiapan tindakan
D.      Observasi dan inperetasi.
SOAL REFLEKSI PEMBELAJARAN 

120. Kegiatan yang harus dilakukan setelah pembelajaran adalah melakukan refleksi.
Refleksi ini tidak hanya dilaksanakan pendidik saja, tetapi juga oleh peserta didik. Tujuan
dilakukannya refleksi pembelajaran bagi peserta didik adalah....
A.   untuk mengidentifikasi faktor-faktor penyebab kegagalan pendidik dalam
pembelajaran
B.    untuk mencapai kepuasaan diri peserta didik memperoleh wadah yang tepat
dalam menjalin komunikasi positif dengan pendidik.
C.   untuk meningkatkan motivasi belajar peserta didik dalam mengikuti pembelajaran
selanjutnya
D.   untuk mencapai kepuasaan diri pendidik memperoleh wadah yang tepat dalam
menjalin komunikasi positif dengan peserta pendidik.

121. Kegiatan yang harus dilakukan setelah pembelajaran adalah melakukan refleksi.
Tujuan dilakukan refleksi pembelajaran bagi pendidik antara lain adalah ….
A.   Untuk menganalisis tingkat keberhasilan proses dan hasil belajar peserta didik
B.   Untuk melakukan evaluasi diri terhadap hasil belajar yang telah dilakukan
C.   untuk mengidentifikasi faktor-faktor penyebab kegagalan pembelajaran
D.  Untuk mengembangkan model pembelajaran sesuai dengan mata pelajaran yang
diampu

122. Kegiatan refleksi merupakan kegiatan terakhir dari pelaksanaan pembelajaran.


Pada kegiatan inilah guru akan dapat mengetahui berhasil tidaknya rencana pelaksanaan
pembelajaran. Kegiatan refleksi dilakukan dengan memperhatikan beberapa prinsip,
diantaranya adalah ....
A.   Hasil penilaian pendidik dijadikan masukan oleh pendidik untuk perbaikan
pembelajaran
B.   Ada kesadaran pendidik untuk meningkatkan kualitas pembelajaran
C.   Penilaian dilaksanakan di akhir pembelajaran
D.   Penilaian dilaksanakan sejak awal pembelajaran sampai akhir pembelajaran 

123. Refleksi terhadap pembelajaran mutlak harus dilakukan oleh pendidik untuk
meningkatkan mutu pembelajaran dan meningkatkan kinerjanya sendiri dengan
memperhatikan beberapa prinsip, di antaranya adalah ….
A.  Penilaian dilaksanakan di akhir pembelajaran
B.   Ada kesadaran pendidik untuk meningkatkan kualitas pembelajaran
C.   Penilaian oleh peserta didik dilakukan dengan sangat kritis
D.Hasil penilaian pendidik dijadikan masukan oleh pendidik untuk perbaikan
pembelajaran

124. Untuk mengetahui keberhasilan belajar peserta didik, baik selama maupun setelah
peserta didik mengikuti pembelajaran tertentu dapat dilihat melalui pengamatan keaktifan
peserta didik dalam bekerjasama atau wawancara tentang kesulitan-kesulitan yang
dihadapi peserta didik selama mengikuti pembelajaran.Pertanyaan-pertanyaan yang dapat
menjelaskan memberikan penjelasan tetang hasil wawancara atau pengamatan
diantaranya adalah ...
A.  Mengapa peserta didik tidak aktif dalam mengikuti pembelajaran?
B.  Bagaimanakah proses pembelajaran yang dilakukan agar efektif?
C.  Apakah kompetensi awal peserta didik untuk mengikuti pembelajaran memadai?
D.   mengapa peserta didik kita memberikan respon negatif atas pelaksanaan
pembelajaran yang kita lakukan
125. Pernyataan berikut yang tidak sesuai dengan konsep refleksi pembelajaran
adalah… ...
A. Refleksi merupakan kegiatan analisis-sintesis, interpretasi, dan eksplanasi terhadap
Semua informasi yang diperoleh dari pelaksanaan pembelajaran.
B. Refleksi merupakan suatu kegiatan yang dilakukan untuk melihat kembali apakah
pembelajaran yang dilaksanakan telah sesuai dengan yang kita rencanakan.
C. Refleksi diperlukan untuk memperoleh gambaran tingkat keberhasilan rencana
pembelajaran yang tertuang dalam Rencana Pelaksnaan Pembelajaran.
D. Refleksi adalah sebuah proses memperbaiki pembelajaran yang telah dilakukan
berdasarkan data yang telah dianalisis dan diinterpretasikan.
Pilihlah salah satu jawaban jawaban yang benar! 

1. Kemampuan bahasa anak mulai berkembang, pemikiran masih statis, belum dapat
berfikir abstrak, dan kemampuan persepsi waktu dan ruang masih terbatas merupakan
perkembangan anak pada tahap ... 

A. Tahap sensorimotorik B. Tahap


praoperasional C. Tahap
operasional konkrit D. Tahap
operasional konkrit 

Jawaban : B 

2. Seorang anak dengan kemampuan menggunakan kata secara efektif baik lisan
(pendongeng, orator, penerjemah, dsb), maupun tertulis/tulisan (sastrawan, penulis
skenario drama/film, fonologi atau bunyi bahasa, semantik atau makna bahasa, dimensi
pragmatik atau penggunaan praktis bahasa) menurut Howard Gardner anak tersebut
mempunyai kecerdasan... 

A. Kecerdasan Bahasa (linguistic) B. Kecerdasan Logika


Matematika (logic-mathematical) C. Kecerdasan Ruang
(spatial) D. Kecerdasan Gerak Tubuh 

Jawaban : A 

3. Untuk mengetahui kemampuan awal peserta didik, seorang pendidik dapat


melakukan ... 

A. pengayaan materi pelajaran B.


refleksi C. latar belakang peserta
didik D. tes awal (pre-test) 

Jawaban : D 

4. Dalam mengidentifikasi kasus kesulitan belajar dapat dilakukan dengan metode


criterion referenced. Yang bukan merupakan tahap metode criterion referenced adalah ... 

A. Menetapkan angka nilai kualitatif minimal yang dapat diterima, misalnya 5,0 atau
6,0. B. Membandingkan prestasi dari setiap siswa dengan angka nilai batas lulus
tersebut. 
C. Menganalisis kemampuan peserta didik D. Menghimpun siswa yang
diduga mengalami kesulitan belajar 

Jawaban : C 

5. Dalam kompetensi pedagogik guru mengusai teori belajar dan prinsip prinsip
pembelajaran yang mendidik. Dalam prinsip pembelajaran yang mendidik guru
harus ... 

A. Melaksanakan Proses Pembelajaran sesuai jadwal B. Menerapkan berbagai


pendekatan, strategi, metode, dan teknik pembelajaran yang mendidik secara kreatif
dalam mata pelajaran yang diampu. C. Menggunakan strategi pembelajaran D. Hanya
menggunakan teknik pembelajaran 

Jawaban : B 
6. Salah satu unsur strategi pembelajaran adalah Mengidentifikasi dan menetapkan
spesifikasi dan kualifikasi hasil (out put) dan sasaran (target) yang harus dicapai, dengan
mempertimbangkan aspirasi dan selera masyarakat yang memerlukannya. (Newman dan
Logan (Abin Syamsuddin Makmun, 2003) Penerapan dari unsur strategi di atas dalam
konteks pembelajaran adalah ... 

A. Menetapkan spesifikasi dan kualifikasi tujuan pembelajaran yakni perubahan profil


perilaku dan pribadi peserta didik. B. Mempertimbangkan dan memilih sistem pendekatan
pembelajaran yang dipandang paling efektif. C. Mempertimbangkan dan menetapkan
langkah-langkah atau prosedur, metode dan teknik pembelajaran. D. Menetapkan norma-
norma dan batas minimum ukuran keberhasilan atau kriteria dan ukuran baku
keberhasilan 

Jawaban : A 

7. Yang ditekankan di dalam strategi pembelajaran interaktif adalah... 

A. membangun inisiatif individu, kemandirian, dan peningkatan diri B.


diskusi dan sharing diantara peserta didik C. pembelajaran didominasi
arahan dari guru D. proses penyampaian materi secara verbal 

Jawaban : B 
8. Pada Kurikulum 2013, penyusunan kurikulum dimulai dengan menetapkan SKL
berdasarkan kesiapan siswa, tujuan pendidikan nasional, dan kebutuhan. Setelah
kompetensi ditetapkan kemudian ditentukan kurikulumnya yang terdiri dari kerangka
dasar kurikulum dan struktur kurikulum. Ada beberapa faktor yang harus dikembangkan
antara lain ...... 

A. Tantangan internal dan Eksternal B.


Responsibility C. Reinforced D. Enriched 

Jawaban : A 

9. Kurikulum 2013, proses pembelajaran menggunakan pendekatan saintifik,


Artinya yaitu ........ 

A. kurikulum harus mengandung hal-hal barusehingga dapat membantu siswa untuk ,


dapat mengembangkan setiap potensi yang dimilikinya B. Menyeleksi nilai dan budaya,
mana yang perlu dipertahankan, dan mana yang harus dimiliki oleh siswa C. Pembelajaran
yang mendorong siswa lebih mampu dalam mengamati, menanya, mengumpulkan
informasi, mengasosiasi/menalar, dan mengomunikasikan D. Mengembangkan
Kompetensi Dasar berdasar pada prinsip akumulatif, saling memperkuat (reinforced) dan
memperkaya (enriched) antar-mata pelajaran 

Jawaban : C 

10. Pengembangan kurikulum pemangku kepentingan ( sttka dilakukan dengan


melibatkan pemangku ( stakeholders ) untuk menjamin relevansi pendidikan dengan
kebutuhan kehidupan, termasuk di dalamnya kehidupan kemasyarakatan, dunia
usaha/industri dan dunia kerja. Pernyataan tersebut diatas sesuai dengan prinsip
pengembangan kurikulum : 

A. Tanggap terhadap perkembangan ilmu pengetahuan, teknologi,dan seni B.


Relevan dengan kebutuhan kehidupan C. Menyeluruh dan berkesinambungan D.
Beragam dan terpadu 

Jawaban : C 
11. Kurikulum dikembangkan berdasarkan prinsip bahwa peserta didik memiliki posisi
sentral untuk mengembangkan kompetensinya agar menjadi manusia yang beriman dan
berakwa kepada Tuhan Yang Maha Esa , berakhlak mulia, sehat,berilmu, cakap, kreatif,
mandiri dan menjadi warga Negara yang demokratis 
serta bertanggung jawab. Pernyataan tersebut diatas sesuai dengan prinsip
pengembangan kurikulum .......... 

A. Tanggap terhadap perkembangan ilmu pengetahuan, teknologi,dan seni B.


Relevan dengan kebutuhan kehidupan C. Menyeluruh dan berkesinambungan D.
Beragam dan terpadu 

Jawaban : D 

12. Upaya guru dalam memberikan kesempatan kepada siswa untuk merefleksikan
pengalaman belajar yang telah dialami yang paling optimal adalah dengan cara ........ 

A. Guru memberikan tes atau pekerjaan rumah setiap akhir pelajaran B. Guru
menggunakan pertanyaan terarah maupun pertanyaan bersifat penelusuransaat
pembelajaran berlangsung C. Guru mewawancarai tiap siswa tentang capaian atau
masalah belajar yang dihadapisiswa dalam belajar D. Guru menugaskan siswa untuk
menuliskan kesulitan siswa dalam belajar 

Jawaban : B 

13. Upaya merancang pengayaan bagi peserta didik yang mencapai ketuntasan
belajar optimal tampak dalam kegiatan guru sebagai berikut ............... 

A. Memberikan tambahan materi berupa sumber ajar dari pengarang yang berbeda B.
Memberikan tes tambahan dengan tingkat kesukaran yang lebih tinggi C. Memberikan
tambahan sumber baaan tambahan yang lebih mendalam dan tingkat variasi yang tinggi
berikut instrument tesnya yang sesuai D. Memberikan tambahan materi berupa sumber
ajar dari penerbit yang berbeda 

Jawaban : C 

14. Yang di maksud dengan penilaian adalah 

A. Pengumpulan dan pengolahan informasi untuk mengukur pencapaian hasil belajar


peserta didik. B. Proses yang di lakukan untuk mengambil keputusan berdasaarkan
evaluasi. C. Membandingkan hasil pengamatan dengan suatu kriteria. D. Proses
interaksi antara peserta didik dan pendidik 

Jawaban : A 
15. Bentuk penilaian yang menghendaki peserta didik menampilkan sikap
menggunakan pengetahuan yang di peroleh dalam pembelajaran dalam
melaksanakan tugas di sebut penilaian 

A. Pre test B. post


test C. Autentik D.
Non Autentik 

Jawaban : C 

16. Pendidik melakukan penilaian kompetensi sikap melalui observasi penilaian diri,
penilaian “teman sejawat” (peer evaluation) oleh peserta didik. Adapun instrumen yang
di gunakan dalam penilaian tersebut adalah : 
A. Skala penilaian (rating scole) B.
rubrik C. jurnal D. Soal tes 

Jawaban : A 

17. Lingkup penilaian hasil belajar mencakup kompetensi beberapa kompetensi yaitu; 

A. kompetensi spiritual, kompetensi sikap. B. kompetensi inti, kompetensi dasar C.


kompetensi sikap spiritual, kompetensi sikap sosial, kompetensi keterampilan,
kompetensi pengetahuan. D. kompetensi sikap sosial 

Jawaban : C 

18. Aspek yang di nilai dalam pembelajaran matetamatika sebagai berikut kecuali ; 

A. Pemahaman konsep B.
Menggunakan grafik C. melakukan
prosedur D. pemecahan masalah dan
sikap 

Jawaban : D 

19. Konsep menyodorkan berbagai situasi kepada siswa dan mendorong siswa untuk
menyelidiki, mencari jawabannya adalah konsep dan strategi..... 

A. Juscovery learning B.
Induktif learning C.
Inquiry learning D.
Cooperatif learning 

Jawaban : C 

20. Guru memebentuk kelompok kelompok belajar dan memeberikan tugas kepada tiap
kelompok . Setiap anggota dari suatu kelompok memberikan tugas dan fungsi tertentu
sehingga jika salah satu tidak menjalankan tugasnya maka pekerjaan kelompok tersbut
tidak dapat di selesaikan . karakteristik dari strategi pembelajaran tersebu adalah ; 

A. Tugas kelompok B. Diskusi kelompok kecil


C. Pengajaran langsung (direct teaching) D.
belajar kooperatf (cooperatf learning) 

Jawaban : A 

21. Teknologi informasi dan komunikasi dalam pembelajaran memiliki tiga fungsi utama
yang di gunakan dalam kegiatan pembelajaran kecuali 

A. Teknologi sebagai alat bantu bagi siswa dalam pembelajaran B.


Teknologi sebagai ilmu pengetahuan (science) C. Teknologi sebagai bahan
dann alat bantu pembelajaran (literacy) D. Teknologi sebagai alat
pengembangan produk 

Jawaban : D 

22. Bu Ani mengajak siswa kelas V ke ladang jagung milik petani untuk mengamati
tanaman jagung yang siap di panen muda. Selanjutnya ibu Ani memeriksa laporan
hasil observasi siswa. Dari hasil klasifikasi kecenderungan gaya belajar siswa di
peroleh peringkat hasil belajar peringkat terbaik dari pembelajaran berupa observasi
di luar kelas menunjukkan siswa memeiliki kecenderungan gaya belajar berhasil baik
melalui..... 

A. Visual (pandangan mata)- Auditif (pendengaran) B.


Visual (pandangan mata ) – Kinestik (gerak ) C. Auditif
(pendengaran )- Kinestetik (gerak) D. Auditif Visual dan
kinestetik 

Jawaban : D 

23. Upaya membimbing siswa untuk mengembang ketrampilan pengetahuan


terlihat dalam upaya guru: 

A. memberi contoh penting toleransi B. mendiskusikan bagaimana mengubah


permasalahan disekitar siswa C. melatih bagaimana mempersiapkan kesehatan diri
dan lingkungan sekitar D. melatih siswa membuat keputusan yang diambil
berdasarkan informasi 

Jawaban : B 

24. Upaya guru menggunakan hasil analisis untuk menentukanketuntasan belajar


antara lain sebagai berikut ......... 

A. menentukan kreteria keberhasilan belajar B. mengklasifikasi siswa berdasarkan hasil


capaian belajarnya C. mencari letak kelemahan secara umum dilihat dari kreteria
keberhasilan yang diharapkan D. merencanakan pengajaran remidi 

Jawaban : D 

25. Kegemaran anak akan dongeng /cerita yang bersifat kritis seperti kisah
perjalanan riwayat para pahlawan dan sebagainya merupakan ciri-ciri anak
berusia......... 

A. 3-5tahun B. 6-8
tahun C. 10-12
tahun D. 15-18
tahun 

Jawaban : C 

26. Beberapa siswa memiliki kecenderungan belajar berupa mengerjakan soal atau tugas
dengan cepat tetapi hasilnya banyak kesalahan .Gaya belajar para siswa tersebut
adalah....... 

A. implusive B. reflektif C.
gaya berfikir terikat D.
gaya berfikir bebas 

Jawaban : A 
27. Hal yang mengandung makna Pancasila sebagai kaidah dasar Negara
bersifat....... A. jujur dan adil B. mengikat dan memaksa C. sementara dan
selamanya D. sebenarnya dan sejujunya 

Jawaban : B 

28. Penilaian yang didasarkan pada data yang mencerminkan kemampuan yang diukur
,hal tersebut merupakan prinsip penilaian yang ...... 
A. Adil B.
Obyektif C. Valid
D. Sistematis 

Jawaban : D 

29. Jika roda pertama berputar 3 kali, dan roda kedua berputar 9 kali,
berapakalikah roda kedua akan berputar jika roda pertama berputar 10 kali 

A. 30 B.
21 C. 27
D. 18 

Jawaban : A 

30. Serangkaian kegiatan yang sistematik untuk dapat menentukan manfaat atau
kegunaan suatu obyek atau program adalah .... 

A. Pengukuran B.
Evaluaasi C.
Penilaian D.
Penilaian 

Jawaban : B 

31. Proses mendengarkan yang dilakukan dengan sungguh-sungguh disebut..... 

A. Membaca intensif B.
Membaca langsung C.
Membaca deduktif 
D. Membaca sekilas 

Jawaban : A 

32. Etika Pancasila adalah ..... 

A. Ketuhanan, kemanusiaan dan keadilan B.


Keagamaan, social dan kerakyatan C. Kebangsaan ,
kedaerahan dan kebudayaan D. Ketuhanan,
kerakyatan dan kebangsaan 

Jawaban : A 

33. Dalam kaitannya dengan upaya untuk memotivasi belajar siswa dan agar proses
pembelajaran berlangsung efektif, maka guru perlu mengacu pada.... 

A. Metode pembelajaran B.
Pendekatan pembelajaran C.
Strategi pembelajaran D. Gaya
pembelajaran 

Jawaban : C 

34. Manfaat Penelitian Tindakan Kelas (PTK) adalah (1) meningkatkan proses belajar
mengajar, (2) menggunakan rancangan yang bersifat siklus di mana setiap siklus
memperhatikan perubahan langkah demi langkah, dan (3) mencakup proses refleksi dan
partisipatif, kecuali.... 
A. Meningkatkan proses belajar mengajar B.
Menggunakan rancangan yang bersifat siklus C.
Mencakup proses refleksi D. Menggunakan
metodelogi terbaru 

Jawaban : D 

35. Pada suatu hari dalam perjalanan menumpangi mobil angkot. Dua penumpang yang
masih muda belia tertawa, tetapi tidak terdengar mereka melakukan interaksi. Karena
penasaran, saya mencoba memperhatiakan apa yang mereka lakukan. Ternyata mereka
adalah siswa-siswa tuna rungu sedang asyik berkomunikasi, akan tetapi komunikasi yang
dilakukan tidak menggunakan bahasa. Mereka menggunakan jari-jari tangan untuk
berkomunikasi. Dengan demikian mereka menggunkan bahasa isyarat. Dari ilustrasi di
atas, bahasa termasuk komunikasi..... 

A. Verbal 
B. Nonverbal C.
Inverbal D.
Sistimatik 

Jawaban : B 

36. Seorang bayi mulai berinteraksi dengan mengeluarkan bunyi-bunyi yang tidak
beraturan ketika diperlihatkan sebuah maianan dan diajak bicara. Seorang ibu seringkali
memberi kesempatan kepada bayi untuk ikut dalam komunikasi sosial dengannya. Bayi
mengangkat-angkat badannya seolah-olah memberi tanda untuk minta digendong. Ketika
itulah bayi pertama kali mengenal sosialisasi dan merasakan bahwa dunia ini tempat
orang saling berbagi rasa. Proses anak mulai mengenal komunikasi dengan
lingkungannya secara verbal disebut dengan.... 

A. Pemerolehan bahasa B.
Pemerolehan isyarat anak C.
Pemecahan bahasa anak D.
Karakteristik anak 

Jawaban : A 

37. Penggunaan huruf kapital di bawah ini yang benar adalah.... 

A. “kapan bapak berangkat?” tanya dirman B. Presiden


jokowi berkunjung ke blora. C. Dari Ave Maria ke Jalan
Lain ke Roma D. teluk bone sungguh indah dan
mengagumkan. 

Jawaban : C 

38. Bahasa Indonesia Baku adalah bahasa Indonesia yang mengikuti kaidah atau pola
bahasa Indonesia yang sedang berlaku. Penggunaan huruf miring pada kalimat di atas
digunakan untuk.... 

A. Menulis judul buku B. Menjelaskan atau mengkhususkan huruf, bagian kata, atau
kelompok kata C. Menuliskan kata nama-nama ilmiah D. Menuliskan nama majalah 

Jawaban : B 

39. Masyarakat Indonesia terdiri dari berbagai suku dengan bahasa yang berbeda- beda,
maka akan sulit berkomunikai kecuali ada satu bahasa pokok yang 
digunakan. Maka dari itu digunakanlah Bahasa Indonesia. Hal ini Bahasa
Indonesia memiliki kedudukan dan fungsi.... 

A. Sebagai lambang kebanggaan kebangsaan B.


Sebagai alat pemersatu bangsa C. Sebagai alat
perhubungan D. Sebagai lambang identitas
nasional 

Jawaban : C 

40. Untuk menunjang kegiatan pembelajaran dan segala hal dalam konteks pendidikan
digunakanlah Bahasa Indonesia. Hal tersebut merupakan fungsi Bahasa Indonesia
sebagai.... 

A. Alat perhubungan tingkat nasional B. Alat pengembangan


kebudayaan dan IPTEK nasional C. Bahasa yang digunakan
dalam peristiwa kenegaraan D. Bahasa pengantar di lembaga
pendidikan 

Jawaban : D 

41. Model pembelajaran yang cocok dengan mata pelajaran matematika adalah.... 

A. Model Inquiry Learning B. Model


Discovery Learning C. Model Problem
Based Learning D. Model Project Based
Learning 

Jawaban : A 

42. Teman-teman, pernah punya pengalaman seperti Fikri, nggak? Sewaktu dia bermain di
sungai bersama teman-temannya, dia mnginjak batu sungai yang licin. Akibatnya dia
terpeleset dan byuurrr... jatuh deh ke sungai. Waah... asyik, ya, bermain di sungai. Eh,
teman-teman tahu nggak, mengapa batu di sungai halus dan licin sedangkan batu-batu di
darat banyak yang kasar? Begini penjelasannya, simak ya.... Dalam bacaan di atas
menggunakan ragam bahasa.... 

A. Resmi B. Baku
C. Tidak baku D.
Daerah 

Jawaban : C 

43. Peggunaan tanda baca yamg benar dibawah ini adalah ............ 

A. Kita sekarang memerlukan meja. kursi. dan lemari. B. Kita,


sekarang memerlukan meja, kursi, dan lemari ! C. Kita
sekarang memerlukan meja, kursi dan lemari. D. Kita
sekarang memerlukan meja kursi dan lemari ? 

Jawaban : C 

44. Menyimak sebagai sebuah keterampilan berbahasa , Karena ......... 

A. Menyimak sambil melakukan aktivitas lain tidak mampu menanggapi secara tepat B.
Menyimak sambil melakukan aktifitas lain mampu menanggapi secara tepat C.
Membaca sambil menyimak mampu menangkap pembicaraan secara benar D.
Menyimak sambil menyanyi , ternyata dapat menanggapi secara tepat 

Jawaban : B 

45. Dalam pembelajaran membaca permulaan, ada beberapa metode yang dapat
digunakan kecuali ............ 

A. Metode Kupas rangkai suku kata B.


Metode kata lembaga C. Metode SAS D.
Metode Cerita 

Jawaban : D 

46. Berikut diberikan studi kasus : Mula – mula diberikan kalimat secara keseluruhan.
Kalimat itu diuraikan atas kata – kata yang mendukungnya. Dari kata – kata itu kita
ceraikan atas suku – suku katanya dan akhirnya atas huruf – hurufnya.Kemudian huruf –
huruf itu kita sintetiskan kembali menjadi suku kata, suku kata menjadi kata dan kata
menjadi kalimat. Berdasarkan studi kasus metode membaca permulaan yang tepat
digunakan adalah ......... 

A. Metode Alfabet B.
Metode Suku Kata C.
Metode SAS D. Metode
Cerita 
Jawaban : B 

47. Memilih berbagai metode menulis permulaan 1). Menulis permulaan 2).
Merangkaikan huruf lepas menjadi suku kata 3). Merangkaikan suku kata
menjadi kata 4). Menyusun kata menjadi kalimat Tahapan diatas adalah teknik
menulis permulaan dengan metode ............ 

A. Metode Eja B. Metode Kata


Lembaga C. Metode SAS D.
Metode Global 

Jawaban : C 

48. Merancang berbagai kegiatan menulis di kelas tinggi Berikut adaah


kegiatan menulis lanjutan di kelas tinggi,kecuali.... 

A. Menulis tentang berbagai topik B.


Menulis pengumuman C. Menulis
pantun D. Menulis memo 

Jawaban : C 

49. Kegiatan mementaskan lakon atau cerita biasanya guru dan siswa mempersiapkan
naskah dan skenario,perilaku dan perlengkapan.kegiatan tersebut mengekspesikan
perasaan dan pikiran siswa dalam bentuk bahasa lisan.hal tersebut termasuk hakekat
berbicara jenis..... 

A. Bercerita B. Cerita
berantai C. Bermain
peran D. Dramatisasi 

Jawaban : D 
50. 1) Warna batunya merah menyala seperti api, bila terkena sinar matahari akan
menyilaukan mata. 2) Ayah memiliki batu akik yang sangat-sangat ia sukai. 3) Batu akik
ini sungguh ajaib karena seakan menyala saat lampu mati. 4) Tidak salah jika Ayah sangat
suka dengan batu akiknya ini. Urutan kalimat-kalimat di atas agar menjadi paragraf yang
padu adalah ... 

A. 1 – 4 – 2 – 3 B. 1
– 3 – 2 – 4 C. 2 – 1 –
3 – 4 D. 2 – 3 – 4 –

Jawaban : C 
Contoh Soal PTK

Soal-soal Latihan berikut ini terdiri dari dua bagian. Bagian pertama adalah soal-soal dalam
bentuk pilihan ganda dan bagian kedua adalah soal-soal dalam bentuk essay.
I. Bagian I (Bentuk pilihan ganda)
Pilihlah alternative jawaban yang paling benar dengan cara memberikan tanda silang (X)
pada option a, b, c, d atau e
1. Apabila dikaitkan dengan tanggung jawab guru terhadap pembelajaran, PTK dapat
membantu guru untuk......................
a. Mengatasi masalah siswa
b. Berkolaborasi dengan guru lain
c. Berkembang secara professional
d. Memperbaiki pembelajaran
e. Mengembangkan kurikulum

2. Pengertian kelas dalam PTK adalah sekelompok peserta didik yang sedang belajar.
Adapun komponen dalam sebuah kelas yang dapat dikaji melalui penelitian
tindakan antara lain sebagai berikut, kecuali:
a. Siswa
b. Guru
c. Materi pelajaran
d. Hasil pembelajaran
e. Sekolah

3. PTK memiliki ciri khusus yaitu sikap reflektif berkelanjutan, artinya bahwa............
a. PTK lebih menekankan pada proses refleksi terhadap proses dan hasil penelitian
guna memperbaiki proses tindakan pada siklus berikutnya
b. PTK merupakan upaya perbaikan proses dan hasil pembelajaran secara
kolaboratif
c. PTK berangkat dari permasalahan pembelajaran riil sehari-hari yang dialami oleh
guru
d. PTK adalah penelitian eksperimen
e. PTK merupakan jenis penelitian yang paling tepat untuk perbaikan proses
pembelajaran

4. Berikut ini yang merupakan judul PTK adalah...........


a. Analisis Tingkat Berpikir Kreatif Siswa Kelas VIII SMP Negeri 30 Surakarta dalam
Menyelesaikan Soal Sistem Persamaan Linier.
b. Pengaruh pembelajaran Berbasis Konstruktivistik dan Kontekstual pada Materi
Pecahan Ditinjau dari Gaya Belajar Siswa Kelas V SD Jaten II.
c. Penerapan Model Pembelajaran Learning Cycle pada Materi Trigonometri di SMA
d. Upaya Peningkatan Kemampuan Pemecahan Masalah Materi Perbandingan pada
Siswa Kelas VII SMP Negeri 2 Karanganyar.
e. Optimalisasi Pembelajaran Matematika dengan Pengorganisasian Tugas
Terstruktur dan Kuis pada Siswa Kelas VIII SMP Negeri 12 Surakarta Tahun
Pelajaran 2009 / 2010

5. Berikut ini yang bukan merupakan rumusan masalah PTK adalah...........


a. Apakah penerapan model pembelajaran Problem Based Instruction dapat
meningkatkan kualitas proses pembelajaran materi pokok permutasi dan
kombinasi di SMA ?
b. Bagaimanakah penerapan model pembelajaran Problem Based Instruction yang
dapat meningkatkan kualitas proses pembelajaran materi pokok permutasi dan
kombinasi di SMA ?
c. Bagaimanakah pengorganisasian tugas terstruktur dan kuis yang dapat
mengoptimalisasikan pembelajaran matematika pada siswa kelas VIII SMP ?
d. Apakah terdapat pengaruh pendekatan pembelajaran Open Ended ditinjau dari
kreativitasbelajar siswa terhadap prestasi belajar matematika pada materi
pokok statistika di SMANegeri 5 Cimahi ?
e. Apakah pengorganisasian tugas terstruktur dan kuis dapat mengoptimalisasikan
pembelajaran matematika pada siswa kelas VIII SMP ?
6. Masalah yang diangkat sebagai masalah dalam PTK adalah masalah tersebut harus
fleksible yang artinya....................
a. Bersifat nyata terjadi di kelas
b. Masalah tersebut dapat dipecahkan
c. Masalah tersebut perlu untuk dipecahkan
d. Mempunyai urgensi jangka pendek yang jelas
e. Dapat membawa keuntungan

7. Cara pemecahan masalah pada PTK ditentukan berdasarkan...........


a. Perumusan hipotesis
b. Identifikasi masalah penelitian
c. Akar penyebab permasalahan dalam bentuk tindakan yang jelas dan terarah
d. Kerangka berpikir penelitian
e. Rumusan masalah

8. Latar belakang masalah merupakan factor penting yang perlu diuraikan pada bagian
pendahuluan. Berikut ini yang bukan merupakan isi dari latar belakang masalah
adalah :
a. Identifikasi alternative tindakan untuk pemecahan masalah
b. Argumentasi logis terhadap pilihan tindakan
c. Menulis kenyataan yang ada (kondisi awal)
d. Adanya masalah yaitu kesenjangan antara kenyataan dan harapan
e. Perumusan dan pemecahan masalah PTK

9. Empat tahapan utama dalam melaksanakan kegiatan PTK adalah................


a. Perencanaan – refleksi – tindakan – observasi
b. Perencanaan – tindakan – refleksi – observasi
c. Perencanaan – tindakan – observasi – refleksi
d. Perencanaan – observasi – tindakan – refleksi
e. Perencanaan – observasi – refleksi – interpretasi

10. Tindakan yang dipilih dalam melaksanakan PTK harus applicable, artinya...........
a. Dapat meningkatkan kemampuan guru dalam mengelola kelas
b. Dapat meningkatkan kemampuan siswa dalam memahami pelajaran
c. Dapat dan mungkin dilaksanakan
d. Mudah dan dapat diukur
e. Dapat menjanjikan hasil yang optimal

11. Keberhasilan pelaksanaan penelitian tindakan kelas dapat diukur dari.............


a. Perkembangan intelegensi siswa yang cukup berarti
b. Adanya perubahan dalam strategi belajar siswa
c. Adanya perbaikan dalam pembelajaran
d. Kemampuan siswa mengatasi masalahnya sendiri
e. Kemampuan guru dalam mengatasi masalah pembelajaran

12. Tahap perencanaan pada siklus I intinya adalah identifikasi masalah dan penetapan
alternative pemecahan masalah. Berikut ini yang bukan merupakan kegiatan pada
tahap tersebut adalah..........................
a. Menyusun dan mengembangkan scenario pembelajaran
b. Melakukan observasi dengan menggunakan format observasi
c. Merencanakan pembelajaran yang akan diterapkan dalam PBM
d. Menyiapkan sumber belajar
e. Mengembangkan format evaluasi dan observasi

13. Pentingnya merumuskan indicator kinerja pada proposal PTK adalah.............


a. Sebagai tolok ukur dalam menentukan keberhasilan atau keefektifan PTK yang
dilakukan
b. Untuk mengetahui jumlah siklus yang perlu dilakukan pada suatu PTK
c. Agar PTK yang dilakukan lebih berkualitas
d. Untuk melihat kinerja guru dalam mengelola kelas
e. Untuk melihat kinerja siswa dalam pembelajaran
14. Kerangka berpikir yang baik antara lain memuat.................
a. Identifikasi masalah PTK
b. Pertautan antar variable yang diteliti dan teori yang mendasarinya
c. Analisis masalah penelitian
d. Analisis alternative tindakan
e. Uraian sistematis tentang kajian teori dan hasil penelitian yang relevan

15. Yang merupakan salah satu kegiatan PTK pada tahap refleksi antara lain :
a. Menentukan pokok materi matematika sebagai focus materi PTK
b. Menyiapkan media pembelajaran
c. Melakukan evaluasi tindakan pada tiap siklus
d. Menyiapkan sumber belajar
e. Menerapkan tindakan mengacu pada scenario pembelajaran pada RPP

II. Bagian II (Bentuk Essay)


Agar anda lebih menguasai tentang PTK khususnya Proposal PTK sebagai output dari
workshop PTK ini, kerjakanlah latihan berikut sesuai dengan masalah pembelajaran matematika
yang real anda alami di kelas.
A. Cobalah identifikasi masalah-masalah pembelajaran matematika yang paling merisaukan di
kelas anda
1. ..................................................
2. ..................................................
3. ..................................................
4. ..................................................
B. Fokus permasalahan PTK
1. ..................................................
2. ..................................................
3. ..................................................
4. ..................................................
C. Diagnosis penyebab permasalahan
1. ..................................................

2. ..................................................
3. ..................................................
D. Alternative tindakan perbaikan pembelajaran
1. ..................................................
2. ..................................................
3. ..................................................
E. Susunlah topic PTK dengan mengingat tiga unsur yang harus dipenuhi suatu judul PTK
.......................................................
F. Susunlah suatu Proposal PTK yang bersesuaian dengan masalah yang anda tetapkan di atas
dengan mengacu pada sistematika yang ditentukan!

KUNCI JAWABAN :

Bagian I (Bentuk Pilihan Ganda)


1. D 6. B 11. C
2. E 7. C 12. B
3. A 8. E 13. A
4. E 9. C 14. B
5. D 10. C 15. C

Bagian II (Bentuk Essay)


Karena soal latihan berbentuk essay, maka peserta dapat mencoba menilai sendiri
berdasarkan pengalaman anda sebagai seorang guru dalam memberikan skor pada tes essay.
Selain itu, dengan mengacu pada uraian, penjelasan, contoh tiap-tiap komponen dan sistematika
proposal PTK pada modul workshop PTK ini maka anda akan dapat menyelesaikan soal latihan
II dengan baik.
SOAL UKG 2013 KD 1.1 
CONTOH SOAL UJI KOMPETENSI GURU 2013 
YANG DISESUAIKAN DENGAN KISI-KISI SOAL UKG 

KOMPETENSI 
1. Menguasai subtansi dan metodologi dasar keilmuan bahasa Indonesia yang mendukung
pembelajaran bahasa Indonesia SD/MI 
SUB KOMPETENSI 
1.1 Memilih, menata dan merepsentasi materi ajar bahasa Indonesia SD berdasarkan
pemahaman tentang bagaimana siswa belajar bahasa Indonesia 
INDIKATOR ESENSIAL 
1.1.1 Menganalisis karakteristik perkembangan bahasa anak usia SD 
1.1.2 Memilih materi ajar aspek membaca di kelas rendah SD 
1.1.3 Memilih materi ajar aspek menulis di kelas tinggi 

CONTOH SOAL 
No Soal 1a (sesuai IE 1.1.1) 
Berikut ini adalah Karakteristik Anak Usia SD dari segi Psikomotorik, yaitu 
a. Anak sudah dapat memakai pakaian dengan rapi 
b. Anak dapat menciptakan sesuatu bentuk/benda dengan menggunakan alat 
c. Anak sudah mulai memahami beberapa konsep abstrak seperti menghitung tanpa
menggunakan benda 
d. Anak menunjukan kepedulian terhadap orang lain 

Jawaban : B 
Pembahasan 
Perkembangan bahasa anak usia SD dapat dilihat dari beberapa segi 
 Mental (cth: Anak mulai menunjukan tenggang rasa dan penghargaan terhadap
teman, Anak menunjukan kepedulian terhadap orang lain) 
 Psikomotorik (cth: Anak mulai menciptakan sesuatu bentuk/benda dengan
menggunakan alat, terjadi perubahan fisik, tinggi dan berat badan yang menonjol,
Anak sudah memiliki gerakan yang bebas dan aman, Anak mampu melakukan
koordinasi dan keseimbangan badan) 
 Sosial (cth: Anak sudah memakai pakaian dengan rapih, Anak dapat menampilkan
sifat ingin tahu) 
 Emosional (cth: Anak menunjukan keceriaan dalam berbagai aktivitas bersama
teman sebayanya, Anak dapat menunjukan sikap marah dalam kondisi yang wajar) 
 Kognitif (cth: Anak menunjukan kreatifitasnya dalam membentuk karya tertentu,
Anak sudah mulai memahami beberapa konsep Abstak seperti menghitung tanpa
menggunakan benda) 

No Soal 1b (sesuai IE 1.1.1) 


Faktor kendala yang mempengaruhi keterampilan berbahasa anak adalah sebagai berikut,
kecuali … 
a. Jenis Kelamin 
b. Keluarga 
c. Keinginan dan dorongan Komunikasi 
d. Kebiasaan 
Jawaban D
Pembahasan
Faktor penyebab perbedaan tersebut: 
 Kesehatan (Anak yang sehat lebih cepat belajar berbicara dibandingkan dengan
anak yang kurang sehat, sebab perkembangan aspek aspek motorik dan aspek
mental berbicaranya lebih baik sehingga lebih siap untuk belajar berbicara) 
 Kecerdasan (Anak yang memiliki kecerdasan tinggi, akan belajar berbicara lebih
baik dan memiliki penguasaan bahasa erat kaitannya dengan kemampuan berpikir) 
 Jenis kelamin (Anak perempuan lebih dalam belajar bahasa daripada anak laki-laki,
baik dalam pengucapan, kosa kata maupun keseringan berbahasa) 
 Keluarga (Semakin banyak jumlah anggota keluarga akan semakin sering anak
mendengar dan berbicara. Demikian pula anak pertama lebih baik perkembangan
berbicaranya karena orang tua lebih banyak memiliki waktu untuk berbicara dan
berbahasa) 
 Keinginan dan Dorongan Komunikasi (Semakin kuat keinginan dan dorongan untuk
berkomunikasi dengan orang lain terutama teman sebaya, akan semakin kuat pula
usaha anak untuk berbicara dan berbahasa)
 Kepribadian (Anak yang dapat menyesuaikan diri dengan baik dan memiliki
kepribadian yang baik cenderung memiliki kemampuan bicara dan berbahasa lebih
baik daripada anak yang mengalami masalah dalam penyesuaian) 
No Soal 2a (Sesuai IE 1.1.2) 
Tahapan proses belajar membaca bagi siswa sekolah dasar kelas awal biasanya
menggunakan pembeajaran membaca… 
a. Ejaan Per ejaan 
b. Skimming 
c. Permulaan 
d. Cepat 
Jawaban C 
Pembahasan 
Sesuai dengan Kurikulum maka pada kelas rendah (kelas 1 – kelas 2) menggunakan
pembelajaran membaca pemulaan dimana tingkatan proses pembelajaran membaca ini
untuk menguasai sistem tulisan sebagai representasi visual bahasa. 

No Soal 2b (Sesuai IE 1.1.2) 


Dalam pembelajaran membantu permulaan, ada beberapa metode yang dapat digunakan
kecuali … 
a. Metode kupas rangkai suku kata 
b. Metode Global 
c. Metode SAS 
d. Metode Eja
Jawaban D 
Pembahasan 
Beberapa metode pembelajaran menulis awal diantaranya… 
 Metode Abjad/Alfabet. Mula-mula guru memperkenalkan huruf (abjad) kepada
siswa: a b c d e f g h i j k l m n o p q r s t u v w x y z. Selain yang dipasang di papan
tulis, masing-masing huruf tadi juga perlu ditulis dalam sebuah kartu (satu huruf
satu kartu) 
 Metode Suara. Metode suara juga disebut :”Phonic Method”. Metode ini merupakan
penyempurnaan metode alphabet. Pada metode ini bukan abjadnya yang di ajarkan,
melainkan bunyi-bunyi bahasa sebagai pengganti huruf-huruf tersebut.Ucapan
huruf-huruf tidak berdasarkan atas bunyi abjadnya melainkan ucapan hurufnya. 
 Metode Kupas-Rangkai Suku Kata. Berbeda dari metode abjad di atas, metode
kupas-rangkai suku kata ini dimulai dengan pengenalan kata terlebih dahulu.
Misalnya: mama. Kita perlu juga menjelaskan arti kata mama itu kepada siswa agar
mereka mendapatkan makna dari apa yang dipelajari 
 Metode Cerita. Pelaksanaan metode cerita dalam mengajarkan membaca permulaan
diawali dengan menghafalkan cerita atau sebuah puisi. Cerita atau puisi itu
diuraikan atas kalimat-kalimtnya sampai pada kata-katanya. Dalam mengucapkan
kata0kata metode ini menggunakan kata-kata fonetik. 
 Metode Global. Menurut Teori Gestalt, suatu kesatuan lebih bermakna daripada
bagian-bagian. Metode global dimulai dengan mengenalkan kalimat utuh kepada
siswa. Contohnya: ibu makan nasi, disertai gambar, anak membaca tulisan tersebut,
baru guru menjelaskan huruf-huruf yang dirangkai membentuk suku kata, kata, dan
kalimat. 
 Metode SAS — Struktural Analisa Sintesa. Metode SAS dilaksanakan dengan
menggunakan kartu kalimat dan papan flanel. Mula-mula guru menunjukkan
gambar kepada siswa (jika benda asli bisa dihadirkan tentunya lebih baik jika benda
asli ditunjukkan terlebih dahulu) Dalam soal diatas ada metode eja, metode eja
adalah metode dalam menulis bukan dalam membaca. 

No Soal 2c (Sesuai IE 1.1.2) 


Mula-mula diberikan kalimat secara keseluruhan. Kalimat itu diuraikan atas kata-kata yang
mendukungnya. Dari kata-kata itu kita ceraikan atas suku-suku katanya dan akhirnya atas
huruf-hurufnya. Kemidian huruf-huruf itu kita sintetiskan kembali menjadi suku kata, suku
kata menjadi kata dan kata menjadi kalimat… Berdasarkan studi kasus , metode membaca
permulaan yang tepat digunakan adalah 
a. Metode Alfabet 
b. Metode Suku Kata 
c. Metode SAS 
d. Metode Global 
Jawaban D 
Pembahasan lihat pembahasan soal 2b 

No soal 3a (Sesuai IE 1.1.3) 


Pendekatan menekankan keterpaduan empat aspek keterampilan berbahasa (menyimak,
berbicara, membaca, dan menulis) dalam pembelajaran adalah salah satu Pendekatan yang
disarankan dalam pembelajaran menulis dalam melaporkan suatu pengamatan yang
disebut 
a. Pendekatan komunikatif  
b. Pendekatan integratif 
c. Pendekatan keterampilan proses 
d. Pendekatan tematis 
Jawaban C 
Pembahasan 
 pendekatan komunikatif tampak pada butir pembelajaran, misalnya:
mendeskripsikan suatu benda, menulis surat, dan membuat iklan
 pendekatan integratif tampak pada butir pembelajaran, misalnya: menceritakan
pengalaman yang menarik, menuliskan suatu peristiwa sederhana, membaca bacaan
kemudian membuat ikhtisar, dan meringkas cerita yang di dengar; 
 Pendekatan keterampilan proses, tampak pada butir pembelajaran, misalnya:
melaporkan hasil kunjungan, menyusun laporan pengamatan, membuat iklan, dan
menyusun kailimat acak menjadi paragraf yang padu 
 pendekatan tematis, tampak pada butir pembelajaran, misatnya: menulis
pengalaman dalam bentuk puisi, dan menyusun naskah sambutan. 

No soal 3b (Sesuai IE 1.1.3) 


Teknik menulis cerita terdiri atas hal-hal sebagai berikut, kecuali 
a. menjawab pertanyaan  
b. membuat kalimat 
c. subtitusi 
d. persuasi 
Jawaban D 
Pembahasan 
Teknik menyusun cerita dapat dilakukan dengan: (a) menjawab pertanyaan, (b)
melengkapai kalimat, (c) memperbaiki susunan kalimat, (d) memperluas kailimat, (e)
subtitusi, (f) transfomtasi, dan (g) membuat kaiimat 

No soal 3c (Sesuai IE 1.1.3) 


Model pembelajaran menulis cerita/cerpen di SD meliputi hal-hal berikut, kecuali… 
a. Menceritakan gambar 
b. Melanjutkan cerita 
c. Menceritakan pengalaman 
d. Mendeskripsikan cerita 
Jawaban D 
Pembahasan 
Model pembelajaran menulis cerita/cerpen dengan 
1. menceritakan gambar 
2. melanjutkan ceria lain 
3. menceitakan mimpi 
4. menceriakan pengalaman 
5. menceritakan cita-cita 
 

SOAL UKG 2013 KD 1.2 


CONTOH SOAL UJI KOMPETENSI GURU 2013 
YANG DISESUAIKAN DENGAN KISI-KISI SOAL UKG 

KOMPETENSI 
1. Menguasai subtansi dan metodologi dasar keilmuan bahasa Indonesia yang mendukung
pembelajaran bahasa Indonesia SD/MI
SUB KOMPETENSI 
1.2 Merencanakan, melaksanakan, mengorganisasi dan mengevaluasi pembelajaran bahasa
Indonesia  
INDIKATOR ESENSIAL
1.2.1 Memilih berbagai metode pembelajaran menulis permulaan yang dapat
mengembangkan kemampuan dan kegemaran menulis siswa
1.2.2 Merancang berbagai kegiatan menulis kelas tinggi yang dapat meningkatkan
kemampuan menulis dan berpikir siswa
1.2.3 Memperjelas perencanaan dan pelaksanaan penilaian dan evaluasi dalam
pembelajaran bahasa dan sastra Indonesia

CONTOH SOAL
No Soal 3a (sesuai IE 1.2.1)
Metode dengan memulai pengajaran membaca dan menulis permulaan dengan membaca
kalimat secara utuh yang ada di bawah gambar. Menguraikan kalimat dengan kata-kata,
menguraikan kata-kata menjadi suku kata disebut dengan metode 
a. Metode Eja 
b. Metode Kata Lembaga 
c. Metode Global 
d. Metode SAS 
Jawaban : C 
Pembahasan 
Beberapa metode dalam menulis permulaan diantaranya yaitu: 
1. Metode Eja Metode eja di dasarkan pada pendekatan harfiah, artinya belajar membaca
dan menulis dimulai dari huruf-huruf yang dirangkaikan menjadi suku kata. Oleh karena
itu pengajaran dimulai dari pengenalan huruf-huruf. Demikian halnya dengan pengajaran
menulis di mulai dari huruf lepas, dengan langka-langkah sebagai berikut: a) Menulis huruf
lepas b) Merangkaikan huruf lepas menjadi suku kata. c) Merangkaikan suku kata menjadi
kata d) Menyusun kata menjadi kalimat. 
2. Metode kata lembaga Metode kata lembaga di mulai mengajar dengan langkah-langkah
sebagai berikut: a) Mengenalkan kata. b) Merangkaikan kata antar suku kata. c)
Menguraikan suku kata atas huruf-hurufnya. d) Menggabungkan huruf menjadi kata. 
3. Metode Global Metode global memulai pengajaran membaca dan menulis permulaan
dengan membaca kalimat secara utuh yang ada di bawah gambar. Menguraikan kalimat
dengan kata-kata, menguraikan kata-kata menjadi suku kata. 
4. Metode SAS Metode SAS menurut (Djuzak,1996:8) adalah suatu pembelajaran menulis
permulaan yang didasarkan atas pendekatan cerita yakni cara memulai mengajar menulis
dengan menampil cerita yang diambil dari dialog siswa dan guru atau siswa dengan siswa.
Teknik pelaksanaan pembelajaran metode SAS yakni keterampilan menulis kartu huruf,
kartu suku kata, kartu kata dan kartu kalimat, sementara sebagian siswa mencari huruf,
suku kata dan kata, guru dan sebagian siswa menempel kata-kata yang tersusun sehingga
menjadi kalimat yang berarti. 
No Soal 3b (sesuai IE 1.2.1)
Suatu pembelajaran menulis permulaan yang didasarkan atas pendekatan cerita yakni cara
memulai mengajar menulis dengan menampil cerita yang diambil dari dialog siswa dan
guru atau siswa dengan siswa disebut dengan metode
a. Metode Eja
b. Metode Kata Lembaga
c. Metode Global
d. Metode SAS
Jawaban : D
Pembahasan lihat pembahasan 3a

No Soal 4a (sesuai IE 1.2.2)


Berikut adalah kegiatan menulis lanjutan di kelas tinggi kecuali
a. Menulis tentang berbagai topic
b. Menulis pengumuman
c. Menulis pantun
d. Menulis memo
Jawaban A
Pembahasan
Teknis menulis tingkat tinggi dikelompokan menjadi 2 yaitu
1. Menulis cerita a) Teknik menyusun kalimat (menjawab pertanyaan, memperbaiki
sususan kalimat, memperbaiki susunan kalimat, subtitusi, transformasi dan membuat
kalimat) b) Teknik memperkenalkan cerita (baca dan tulis, simak dan tulis, meniru model,
menyusun paragraph, menceritakan kembali, dan membuat) 
2. Menulis untuk keperluan sehari-hari a) menulis surat b) menulis pengumuman c)
mengisi formulir d) menulis surat undangan e) membuat iklan f) menyusun daftar riwayat
hidup 
Model menulis cerita/cerpen di kelas tingkat tinggi yaitu: 
1. menceritakan gambar 
2. melanjutkan cerita lain 
3. menceritakan mimpi 
4. menceritakan pengalaman 
5. Menceritakan cita-cita 
Jenis tulisan yang bisa dikembangkan pada kegiatan menulis lanjutan ini adalah menulis
pantun, puisi, surat dan prosa 

No Soal 4b (sesuai IE 1.2.2)


Model menulis cerita/cerpen di kelas tinggi yaitu, kecuali…
a. Menceritakan mimpi
b. Menceritakan pengalaman
c. Menceritakan cita-cita
d. Menceritakan kata
Jawaban D
Pembahasan lihat pembahasa 4a

No Soal 5a (sesuai IE 1.2.3)


Perencanaan pengajaran meliputi hal-hal berikut, kecuali
a. Tujuan apa yang hendak dicapai
b. Memilih bahan aja
c. Proses belajar mengajar
d. Alat penilaian
Jawaban C
Pembahasan
Perencanaan yang baik harus meliputi
1. Tujuan apa yang hendak dicapai
2. Memilih bahan ajar
3. Memilih metode pengajaran
4. Alat penilaian
No Soal 5b (sesuai IE 1.2.3)
Tahap mengumpulkan informasi tentang keadaan objek evaluasi (siswa) dengan
menggunakan teknik tes atau nontes disebut tahapan 
a. Tahap tindak lanjut
b. Tahap persiapan
c. Tahap pelaksanaan
d. Tahap pengolahan hasil
Jawaban C
Pembahasan
Tahapan dalam evaluasi pembelajaran yaitu
1. Tahap persiapan (tahap untuk menentukan objek evaluasi akan dievaluasi menggunakan
metode apa? Tes atau non tes. Teknik tes yaitu teknik evaluasi dengan memberikan tes
kepada siswa seperti tes tertulis, tes lisan, angket, wawancara. Teknik non tes biasanya
dilihat dari observasi langsung) 
2. Tahap pelaksanaan (tahat mengumpulkan informasi tentang objek evaluasi baik
menggunakan teknik tes maupun non tes) 
3. Tahap pengolahan hasil (tahap pengolahan hasil tes maupun non tes menjadi untuk
ditindaklanjuti hasilnya) 
4. Tahap Tindak lanjut evaluasi (tahap ini biasanya tindak lanjut hasil evaluasi berupa
remedial atau pengayaan)

SOAL UKG 2013 KD 1.3 


CONTOH SOAL UJI KOMPETENSI GURU 2013 
YANG DISESUAIKAN DENGAN KISI-KISI SOAL UKG 

KOMPETENSI
1. Menguasai subtansi dan metodologi dasar keilmuan bahasa Indonesia yang mendukung
pembelajaran bahasa Indonesia SD/MI
SUB KOMPETENSI
1.3 Menampilkan keterampilan berbahasa (mendengarkan, berbicara, membaca dan
menulis)
INDIKATOR ESENSIAL 
1.3.1 Merumuskan hakikat (pengertian, tujuan, jenis dan manfaat membaca)
1.3.2 a) Menemukan isi atau pokok wacana dari sebuah pengumuman b) merumuskan
hakikat (pengertian, tujuan, jenis dan manfaat) menulis
1.3.3 Menemukan isi atau pesan pokok wacana dari sebuah berita
1.3.4 Menemukan isi atau pesan pokok wacana naratif secara cerita rakyat, puisi
1.3.5 Membandingkan berbagai jenis wacana bahasa Indonesia (deskripsi dan narasi)
1.3.6 Membandingkan berbagai wacana Bahasa Indonesia (argumentasi dan eksposisi)
1.3.7 Menyusun berbagai bentuk/jenis tulisan surat
1.3.8 Mendeskripsikan unsur-unsur makalah

CONTOH SOAL
No Soal 7a (sesuai IE 1.3.1)
Membaca yang mengutamakan isi bacaan sebagai ungkapan pikiran, perasaan, dan
kehendak penulis. Bila hanya ingin mengetahui isinya, membaca cerdas bersifat lugas.
Akan tetapi, bial maksudnya untuk memahami dan memilki isi bacaan, maka tergolong
kedalam membaca jenis
a. Membaca cerdas atau membaca dalam hati
b. Membaca bahasa
c. Membaca teknis
d. Membaca bebas
Jawaban A
Pembahasan Jenis-jenis membaca
1) Membaca bahasa Membaca bahasa adalah membaca yang mengutamakan bahasa
bacaan. Membaca bahasa mementingkan segi bahasa bacaan. Hal-hal yang perlu
diperhatikan dalam membaca bahasa adalah kesesuian pikir dengan bahasa,
perbendaharaan bahasa yang meliputi kosa kata, struktur kalimat, dan ejaan. 
2) Membaca cerdas atau membaca dalam hati Membaca cerdas adalah membaca yang
mengutamakan isi bacaan sebagai ungkapan pikiran, perasaan, dan kehendak penulis. Bila
hanya ingin mengetahui isinya, membaca cerdas bersifat lugas. Akan tetapi, bila
maksudnya untuk memahami dan memilki isi bacaan, maka disebut membaca belajar. 
3) Membaca teknis Membaca teknis adalah membaca dengan mengarahkan bacaan secara
wajar. Wajar maksudnya sesuai ucapan, tekanan, dan intonasinya. Pikiran, perasaan, dan
kemauan yang tersimpan dalam bacaan dapat diaktualisasikan dengan baik. 
4) Membaca emosional Membaca emosional adalah membaca sebagai sarana untuk
memasuki perasaan, yaitu keindahan isi, dan keindahan bahasanya. 
5) Membaca bebas Membaca bebas adalah membaca sesuatu atas kehendak sendiri tanpa
adanya unsur paksaan dari luar. Unsur dari luar misalnya guru, orang tua, teman, atau
pihak-pihak lain 

No Soal 7b (sesuai IE 1.3.1)


Membaca jenis ini biasanya dilakukan seseorang membaca demi kesenangan, membaca
bacaan ringan yang mendatangkan kesenangan, kegembiraan sebagai pengisi waktu
senggang. Berdasarkan karakteristik diatas, kegiatan tersebut termasuk ke dalam
membaca jenis
a. Mebaca Survai
b. Membaca Sekilas
c. Membaca Dangkal
d. Membaca Nyaring
Jawaban C
Pembahasan
Jenis membaca berdasarkan karakteristiknya
A. Membaca Nyaring Membaca nyaring adalah kegiatan membaca dengan menyuarakan
tulisan yang dibacanya dengan ucapan dan intonasi yang tepat agar pendengar dan
pembaca dapat menangkap informasi yang disampaikan oleh penulis, baik yang berupa
pikiran, perasaan, sikap, ataupun pengalaman penulis. 

B. Membaca Dalam Hati Membaca dalam hati adalah kegiatan membaca yang dilakukan
dengan tanpa menyuarakan isi bacaan yang dibacanya. 

I. Membaca Ekstensif membaca ekstensif adalah membaca secara luas. Objeknya meliputi
sebanyak mungkin teks dalam waktu yang sesingkat-singkatnya. Membaca ekstensif
meliputi a. Membaca Survai (Survey Reading) Membaca survai adalah kegiatan membaca
untuk mengetahui secara sekilas terhadap bahan bacaan yang akan dibaca lebih mendalam.
Kegiatan membaca survai merupakan pendahuluan dalam membaca ekstensif. b. Membaca
Sekilas Membaca sekilas atau membaca cepat adalah kegiatan membaca dengan
mengandalakan kecepatan gerak mata dalam melihat dan memperhatikan bahan tertulis
yang dibacanya dengan tujuan untuk mendapatkan informasi secara cepat. c. Membaca
Dangkal (Superficial Reading) Membaca dangkal pada hakekatnya bertujuan untuk
memperoleh pemahaman yang dangkal yang bersifat luaran, yang tidak mendalam dari
suatu bahan bacaan. Membaca jenis ini biasanya dilakukan seseorang membaca demi
kesenangan, membaca bacaan ringan yang mendatangkan kesenangan, kegembiraan
sebagai pengisi waktu senggang. 

II. Membaca Intensif Membaca intensif atau intensive reading adalah membaca dengan
penuh penghayatan untuk menyerap apa yang seharusnya kita kuasai. Yang termasuk
dalam membaca intensif adalah 

a. Membaca Telaah Isi 


1. Membaca Teliti Membaca jenis ini sama pentingnya dengan membaca sekilas, maka
sering kali seseorang perlu membaca dengan teliti bahan-bahan yang disukai. 
2. Membaca Pemahaman Membaca pemahaman (reading for understanding) adalah sejenis
membaca yang bertujuan untuk memahami tentang standar-standar atau norma-norma
kesastraan (literary standards), resensi kritis (critical review), dan pola-pola fiksi (patterns
of fiction). 
3. Membaca Kritis Membaca kritis adalah kegiatan membaca yang dilakukan secara
bijakasana, mendalam, evaluatif, dengan tujuan untuk menemukan keseluruhan bahan
bacaan, baik makna baris-baris, makna antar baris, maupun makna balik baris. 
4. Membaca Ide Membaca ide adalah sejenis kegiatan membaca yang ingin mencari,
memperoleh, serta memanfaatkan ide-ide yang terdapat pada bacaan. 
5. Membaca Kreatif Membaca kreatif adalah kegiatan membaca yang tidak hanya sekedar
menagkap makna tersurat, makna antar baris, tetapi juga mampu secara kreatif
menerapkan hasil membacanya untuk kehidupan sehari-hari. 

b. Membaca Telaah Bahasa : 


1. Membaca Bahasa (Foreign Language Reading) Tujuan utama membaca bahasa adalah
memperbesar daya kata (increasing word power) dan mengembangkan kosakata
(developing vocabulary) 
2. Membaca Sastra (Literary Reading) Dalam membaca sastra perhatian pembaca harus
dipusatkan pada penggunaan bahasa dalam karya sastra. Apabila seseorang dapat
mengenal serta mengerti seluk beluk bahasa dalam suatu karya sastra maka semakin
mudah dia memahami isinya serta dapat membedakan antara bahasa ilmiah dan bahasa
sastra. 

No Soal 7c (sesuai IE 1.3.1) 


Berikut adalah tujuan umum dalam aktifitas membaca, kecuali 
a. Membaca untuk memperoleh perincian-perincian atau fakta-fakta 
b. Membaca untuk mengetahui ukuran atau sususan organisasi cerita 
c. Membaca untuk menyimpulkan atau membaca inferensi 
d. Membaca untuk memperoleh kekurangan suatu buku 
Jawaban D 
Pembahasan 
Tujuan membaca antara lain: 
a. Membaca untuk memperoleh perincian-perincian atau fakta-fakta (reading for details or
facts). Membaca tersebut bertujuan untuk menemukan atau mengetahui penemuan-
penemuan telah dilakukan oleh sang tokoh, untuk memecahkan masalah-masalah yang
dibuat oleh sang tokoh. 
b. Membaca untuk memperoleh ide-ide utama (reading for main ideas). Membaca untuk
mengetahui topik atau masalah dalam bacaan. Untuk menemukan ide pokok bacaan
dengan membaca halamn demi halaman. 
c. Membaca untuk mengetahui ukuran atau susunan, organisasi cerita (reading for
sequenceor organization). Membaca tersebut bertujuan untuk mengetahui bagian-bagian
cerita dan hubungan antar bagian-bagian cerita. 
d. Membaca untuk menyimpulkan atau membaca inferensi (reading for inference).
Pembaca diharapkan dapat merasakan sesuatu yang dirasakan penulis. 
e. Membaca untuk mengelompokkan atau mengklasifikasikan (reading for classify).
Membaca jenis ini bertujuan untuk menemukan hal-hal yang tidak wajar mengenai sesuatu
hal (Anderson dalam Tarigan 1979:10). 
f. Membaca untuk menilai atau mengevaluasai (reading to evaluate). Jenis membaca
tersebut bertujuan menemukan suatu keberhasilan berdasarkan ukuran-ukuran tertentu.
Membaca jenis ini memerlukan ketelitian dengan membandingkan dan mengujinya
kembali. 
g. Membaca untuk memperbandingkan atau mempertentangkan (reading to compare or
contrast). Tujuan membaca tersebut adalah untuk menemukan bagaimana cara, perbedaan
atau persamaan dua hal atau lebih 

No Soal 8a (sesuai IE 1.3.2)


Sehubungan dengan pelaksanaan study tour ke Bandung maka diberitahukan kepada
seluruh siswa kelas VI SDN Mande 1 untuk berkumpul pada pukul 19.00 WIB. Dimohon
dengan tepat waktu. Jangan lupa bawa semua perlengkapan seperti Kamera untuk
mendokumentasikan kegiatan dan alat tulis serta pakaian untuk keperluan. Berikut adalah
isi dari pengumuman diatas kecuali 
a. Pengumuman dibuat untuk siswa SDN Mande 1 
b. Peserta study tour berkumpul pada pukul 19.00 WIB 
c. Salah satu perlengkapan yang harus dibawa adalah Kamera 
d. Tujuan study tour tersebut ke Bandung 
Jawaban A 
Pembahasan 
Dalam membaca pengumuman baca dengan teliti pengumuman tersebut. Pada soal kenapa
A salah karena pengumuman diuat untuk siswa kelas VI SDN Mande 1 bukan kepada
seluruh siswa. 

No Soal 8b (sesuai IE 1.3.2) 


Penulis bertujuan untuk menyenangkan para pembaca,menghindarkan kedukaan para
pembaca,ingin menolong para pembaca memahami,menghargai perasaan dan
penalarannya. Penulis harus berkeyakinan bahwa pembaca adalah "teman" hidupnya.
Tujuan penulis tersebut merupakan tujuan menulis sebagai 
a. Tujuan penugasan 
b. Tujuan altruistik 
c. Tujuan persuasif 
d. Tujuan informatif 
Jawaban B 
Pembahasan 
Tujuan menulis sebagai berikut : 
1. Assignment purpose ( tujuan penugasan ). Penulis tidak memiliki tujuan,untuk apa dia
menulis.Penulis hanya menulis karena mendapat tugas,bukan atas kemauan sendiri. 
2. Altruistic purpose ( tujuan altruistik ) Penulis bertujuan untuk menyenangkan para
pembaca,menghindarkan kedukaan para pembaca,ingin menolong para pembaca
memahami,menghargai perasaan dan penalarannya.Penulis harus berkeyakinan bahwa
pembaca adalah "teman" hidupnya. 
3. Persuasive purpose ( tujuan persuasif ) Mendorong atau menarik perhatian pembaca
agar mau menerima informasi yang disampaikan oleh penulis/mempengaruhi pembaca. 
4. Informational purpose ( tujuan informatif/penerangan ). Memberikan informasi atau
keterangan kepada pembaca. 
5. Self expressive purpose ( tujuan pernyataan diri ) Penulis berusaha untuk
memperkenalkan atau menyatakan dirinya sendiri kepada para pembaca. 
 6. Creative purpose ( tujuan kreatif/artistik ) Penulis bertujuan agar para pembaca dapat
memiliki nilai-nilai artistik/keindahan,terharu dengan membaca tulisan si penulis 
7. Problem solving purpose ( tujuan pemecahan masalah ) Penulis berusaha memecahkan
suatu masalah yang dihadapi 

No Soal 9a (sesuai IE 1.3.3)


Hebat, Siswa Indonesia Pertahankan Tradisi Emas di Olimpiade Fisika!
Tim Olimpiade Fisika Indonesia REPUBLIKA.CO.ID, JAKARTA--Luar biasa. Lima siswa
Indonesia yang dikirim ke ajang Olimpiade Fisika atau International Physics Olympiad
(IPhO) ke-41 di Zagreb, Kroasia, 17-25 Juli, berhasil menyabet empat medali emas dan satu
perak. Pelajar yang menyumbang emas adalah Muhammad Sohibul Maromi (SMAN 1
Pamekasan, Madura), Christian George Emor (SMA Lokon St. Nikolaus Tomohon, Sulawesi
Utara), David Giovanni (SMAK Penabur Gading Serpong, Banten), dan Kevin Soedyatmiko
(SMAN 12, Jakarta). Sedangkan medali perak berhasil diraih oleh Ahmad Ataka Awwalur
Rizqi (SMAN 1, Yogyakarta). Prestasi ini jauh lebih baik dibanding ajang Olimpiade Fisika
ke-40 di Merida Yucatan, Meksiko, 2009 yang lalu. Saat itu, delegasi siswa Indonesia
merebut satu medali emas, dua medali perak, dan satu perunggu. Hasil empat medali emas
dan satu perak ini, hampir menyamai prestasi terbaik sebelumnya pada ajang Olimpiade
Fisika ke-37 di Singapura. Saat itu siswa Indonesia tidak hanya berhasil menyabet 4 medali
emas, namun juga meraih predikat `Absolute Winner` atas nama Mailoa Jonathan Pradana
(SMAK 1 BPK Penabur Jakarta). Tapi yang terpenting lagi, pelajar Indonesia berhasil
mempertahankan tradisi emas di setiap ajang Olimpiade Fisika. Red: Endro Yuwanto 
Peristiwa yang diberitakan adalah 
a. Siswa Indonesia pertahankan tradisi emas olimpiade matematika 
b. Siswa Indonesia pertahankan tradisi emas di olimpiade fisika 
c. Siswa Indonesia menyumbang emas di olimpiade matematika 
d. Siswa Indonesia menyumbang emas di olimpiade matematika 
Jawabab B 
Pembahasan Biasanya isi suatu berita tercermin dari judul oleh karena itu yang paling
cocok adalah B No 

Soal 9b (sesuai IE 1.3.3)


Lihat berita di no 9a
Lima siswa Indonesia berhasil mendapatkan
a. 4 emas, 1 perak
b. 1 emas, 4 perak
c. 3 emas, 2 perak
d. 2 emas, 3 perak
Jawaban A
Pembahasan: Baca isi beritanya

No Soal 10a (sesuai IE 1.3.4)


Alkisah, ada sebuah kerajaan bernama Kerajaan Linggar dipimpin oleh seorang ratu,
bernama Ratu Sima. Ratu Sima mempunyai tiga orang putra, semuanya laki-laki. Putra
pertama dan keduanya lahir kembar, bernama Pangeran Seta dan Pangeran Seto. Keduanya
putra kesayangan Ratu Sima. Sementara putra ketiganya lahir cacat. Matanya buta. Ia
bernama Pangeran Rasi. Sejak kecil, Pangeran Rasi dibesarkan oleh Mbok Tumi, dayang
kerajaan yang ditugaskan mengasuh Pangeran Rasi. Mereka hidup bahagia, hingga
Pangeran Rasi menjadi pemuda yang baik dan santun budi pekertinya. Kemarau terus
melanda Kerajaan Linggar. Seluruh rakyat sangat berharap turunnya hujan. Kerajaan
Linggar kekeringan. Ada yang mengatakan, Kerajaan Linggar terkena hukuman karena
keserakahan Pangeran Seta dan Pangeran Seto yang menebang pohon di hutan Randualas,
yaitu hutan yang tumbuh di samping Kerajaan Linggar. Seluruh rakyat tak ada yang berani
melaporkan keserakahan kedua pangeran kembar itu. Semua orang takut karena keduanya
putra kesayangan ratu. (Dikutip dari Cermat Berbahasa, Penerbit Tiga Serangkai) 
Amanat yang dapat kita petik dari cerita tersebut adalah
a. Kita tidak boleh menyombongkan kelebihan yang kita miliki
b. Kita tidak boleh memamerkan kekurangan kita
c. Kesabaran dan ketabahan hati akan mendatangkan kebaikan
d. Kita harus membanggakan diri sendiri supaya percaya diri
Jawaban C
Pembahasan baca wacana naratif

No Soal 10b (sesuai IE 1.3.4) 


Unsur-unsur penting dalam sebuah narasi adalah
a. Kejadian
b. Tokoh
c. Alur
d. Penulis
Jawaban D
Pembahasan
Unsur-unsur penting dalam sebuah narasi adalah: • kejadian, • tokoh, • konflik, • amanat •
alur/plot, • latar yang terdiri atas latar waktu, tempat, dan suasana.

No Soal 11a (sesuai IE 1.3.5) 


Karangan narasi yang berusaha untuk memberikan suatu maksud tertentu, menyampaikan
suatu amanat terselubung kepada para pembaca atau pendengar sehingga tampak seolah-
olah melihat disebut 
a. Narasi Informatif 
b. Narasi Ekspositori 
c. Narasi Sugestik 
d. Narasi Artistik 
Jawaban C 
Pembahasan Jenis-Jenis Wacana Narasi 
a. Narasi Ekspositorik (Narasi Teknis) Narasi ekspositorik adalah narasi yang memiliki
sasaran penyampaian informasi secara tepat tentang suatu peristiwa dengan tujuan
memperluas pengetahuan orang tentang kisah seseorang. Dalam narasi ekspositorik,
penulis menceritakan suatu peristiwa berdasarkan data yang sebenarnya. Pelaku yang
ditonjolkan biasanya satu orang. Pelaku diceritakan mulai dari kecil sampai saat ini sampai
terakhir dalam kehidupannya. Karangan narasi ini diwarnai oleh eksposisi, maka
ketentuan eksposisi juga berlaku pada penulisan narasi ekspositorik. Ketentuan ini
berkaitan dengan penggunaan bahasa yang logis, berdasarkan fakta yang ada, tidak
memasukan unsur sugestif atau bersifat objektif. 
b. Narasi Sugestif Narasi sugestif adalah narasi yang berusaha untuk memberikan suatu
maksud tertentu, menyampaikan suatu amanat terselubung kepada para pembaca atau
pendengar sehingga tampak seolah-olah melihat. 

No Soal 11b (sesuai IE 1.3.5) 


Pendekatan untuk mendapat tanggapan emosional pembaca ataupun kesan pembaca
adalah contoh Pendekatan Deskripsi jenis 
a. Pendekatan Ekspositori 
b. Pendekatan Impresionistik 
c. Pendekatan menurut Sikap Pengarang 
d. Pendekatan Realistik 
Jawaban B 
Pembahasan 
Jenis-Jenis Pendekatan deskripsi 
1. Pendekatan Ekspositoris. Kita berusaha agar deskripsi yang kita buat dapat memberi
keterangan sesuai dengan keadaan yang sebenarnya sehingga pembaca dapat seolah-olah
ikut melihat atau merasakan objek yang kita deskripsikan.Berisi daftar detail sesuatu
secara lengkap atau agak lengkap sehingga pembaca dengan penalarannya dapat
memperoleh kesan keseluruhan tentang sesuatu.Efek pemerolehan kesan tersebut lebih
banyak didasarkan atas proses penalaran daripada emosional. 
2. Pendekatan Impresionistik Untuk mendapatkan tanggapan emosional pembaca ataupun
kesan pembaca.Corak deskripsi ini diantaranya juga ditentukan oleh macam kesan apa
yang diinginkan penulisnya. 
3. Pendekatan menurut sikap pengarang Pendekatan ini sangat bergantung kepada tujuan
yang ingin dicapai,sifat objek serta pembaca deskripsinya.Dalam menguraikan sebuah
gagasan,penulis mungkin mengharapkan agar pembaca merasa tidak puas terhadap suatu
tindakan atau keadaan.Atau penulis menginginkan agar pembaca juga harus merasakan
bahwa persoalan yang tengah dihadapi merupakan masalah yang gawat.Penulis juga dapat
membayangkan bahwa akan terjadi sesuatu yang tidak diinginkan sehingga pembaca dari
mula sudah disiapkan dengan sebuah perasaan yang kurang enak,seram,takut dan
sebagainya. 

No Soal 11c (sesuai IE 1.3.5)


1. Menentukan tema atau amanat apa yang akan disampaikan.
2. Menetapkan sasaran pembaca.
3. Merancang peristiwa-peristiwa utama yang akan ditampilkan dalam bentuk skema
perkembangan,dan akhir cerita.
4. Memerinci peristiwa-peristiwa utama ke dalam detail-detail peristiwa sebagai
pendukung cerita
5. Menyusun tokoh dan perwatakan,serta latar dan sudut pandang.
Karakteristik diatas adalah pengembengan dari paragraph
a. Narasi
b. Deskripsi
c. Ekspositori
d. Sudut Pandang Persuasif
Jawabab A
Pembahasan ke 5 karakteistik merupakan ciri wacana narasi

No Soal 11d (sesuai IE 1.3.5)


1. Tentukan objek atau tema yang akan dideskripsikan
2. Mengumpulkan data dengan mengamati objek yang akan dideskripsikan.
3. Menyusun data tersebut ke dalam urutan yang baik (menyusun kerangka karangan)
4. Menguraikan kerangka karangan menjadi dekripsi yang sesuai dengan tema yang
ditentukan.
5. Tentukan tujuan
Langkah menyusun paragraf deskripsi yang benar adalah
a. 1-5-2-3-4
b. 1-5-4-3-2
c. 1-5-3-2-4
d. 1-3-4-2-5
Jawaban A
Pembahasan langkah-langkah menyusun paragraph deskripsi adalah 1-5-2-3-4

No Soal 12a (sesuai IE 1.3.6)


Berikut ini yang berhubungan dengan paragraph eksposisi, kecuali
a. Memaparkan definisi atau pengertian
b. Memaparkan langkah-langkah melaksanakan suatu kegiatan
c. Menyampaikan informasi
d. Mengajak pembaca untuk menerima papacan tersebut
Jawaban D
Pembahasan
Paragraf eksposisi adalah paragraf yang bertujuan untuk memaparkan, menjelaskan,
menyampaikan informasi, mengajarkan, dan menerangkan sesuatu tanpa disertai ajakan
atau desakan agar pembaca menerima atau mengikutinya. Ciri-ciri paragraf eksposisi: 1)
Memaparkan definisi (pengertian). 2) Memaparkan langkah-langkah, metode, atau cara
melaksanakan suatu kegiatan. 

No Soal 12b (sesuai IE 1.3.6)


Berikut ini yang berhubungan dengan paragraph argumentasi, kecuali
a. Mengungkapkan ide atau gagasan
b. Menjelaskan pendapat agar pembaca yakin
c. Menggali sumber ide dari pengamatan pengalaman dan penelitian
d. Penutup biasanya berisi penegasan tentang ide atau gagasan
Jawaban D
Pembahasan
Karangan argumentasi adalah jenis paragraf yang mengungkapkan ide, gagasan, atau
pendapat penulis dengan disertai bukti dan fakta (benar-benar terjadi). Tujuannya adalah
agar pembaca yakin bahwa ide, gagasan, atau pendapat tersebut adalah benar dan terbukti.
Ciri-ciri karangan argumentasi: 1) Menjelaskan pendapat agar pembaca yakin. 2)
Memerlukan fakta untuk pembuktian berupa gambar/grafik, dan lain-lain. 3) Menggali
sumber ide dari pengamatan, pengalaman, dan penelitian. 4) Penutup berisi kesimpulan. 

No Soal 13a (sesuai IE 1.3.7)


Berikut ini ciri surat pribadi, kecuali
a. Penggunaan bahasa bebas sesuai dengan keinginan penulis
b. Format surat bebas
c. Memiliki kepala surat
d. Memiliki penutup surat
Jawaban C
Pembahasan
Surat pribadi adalah surat yang dipergunakan untuk kepentingan pribadi. Isi surat
berhubungan dengan urusan pribadi. Contohnya surat seorang anak kepada orang tuanya
atau surat kepada teman. Ciri-ciri surat pribadi seperti berikut. (1) Tidak menggunakan
kop surat/kepala surat (2) Tidak menggunakan nomor surat (3) Salam pembuka dan
penutup surat bervariasi (4) Penggunaan bahasa bebas, sesuai dengan keinginan si penulis
surat. (5) Format surat bebas 

No Soal 13b (sesuai IE 1.3.7) 


Dalam surat resmi nomor surat termasuk dalam 
a. Kepala surat 
b. Isi surat 
c. Penutup surat 
d. Badan surat 
Jawaban A 
Pembahasan 
Bagian-Bagian Surat Undangan 
a. Kepala Surat 
(1) nama badan usaha, (2) alamat badan usaha, (3) nomor telepon, (4) nomor kotak pos,
(5) identitas lainnya, (6) tanggal surat, (7) nomor yang ditujukan/alamat dalam. 
b. Isi Surat 
(1) salam pembuka, (2) alasan, (3) hari dan tanggal, (4) waktu, (5) tempat, (6) acara. c.
Penutup/Kaki Surat (1) nama badan usaha, (2) jabatan, (3) nama jelas, (4) nomor induk
pegawai, (5) tembusan. 

No Soal 13c (sesuai IE 1.3.7) 


Surat resmi yang digunakan oleh perusahaan atau badan usaha menurut kepentingan dan
pengirimnya termasuk kedalam jenis surat
a. Surat pribadi 
b. Surat dinas pemerintah 
c. Surat niaga 
d. Surat sosial 
Jawaban C 
Pembahasan 
Jenis-jenis surat menurut kepentingan dan pengirimnya 
a. Surat pribadi,yaitu surat yang dikirimkan seseorang kepada orang lain atau suatu
organisasi/instansi. 
b. Surat dinas pemerintah,yaitu surat resmi yang digunakan instansi pemerintah untuk
kepentingan administrasi pemerintahan. 
c. Surat niaga,yaitu surat resmi yang digunakan oleh perusahaan atau badan usaha. 
d. Surat sosial,yaitu surat resmi yang digunakan oleh organisasi kemasyarakatan yang
bersifat nirlaba ( nonprofit ) 

No Soal 14a (sesuai IE 1.3.8) 


Bagian-bagian berikut ini merupakan bagian dari makalah, kecuali 
a. Pendahuluan 
b. Hipotesis 
c. Pembahasan 
d. Kesimpulan dan Saran 
Jawaban B 
Pembahasan 
Berikut adalah bagian dalam makalah 
1. Pendahuluan berisi (latar belakang, tujuan pembuatan makalah, dan manfaat makalah 
2. Pembahasan 
3. Kesimpulan dan Saran

No Soal 14b (sesuai IE 1.3.8) 


Bagian makalah yang memberikan rekomenadsi kepada pembaca tentang masukan untuk
permasalahan yang dibahas 
a. Latar belakang 
b. Tujuan makalah 
c. Kesimpulan 
d. Saran 
Jawaban D 
Pembahasan 
Latar belakang berisi tentang alasan kenapa makalah ini di buat 
Tujuan makalah adalah untuk apa makalah itu di buat 
Kesimpulan adalah rangkumah hasil pembahasan makalah 
Saran adalah rekomenadsi kepada pembaca tentang masukan untuk permasalahan yang
dibahas 
 

SOAL UKG 2013 KD 1.4 


CONTOH SOAL UJI KOMPETENSI GURU 2013
YANG DISESUAIKAN DENGAN KISI-KISI SOAL UKG 
KOMPETENSI 
1. Menguasai subtansi dan metodologi dasar keilmuan bahasa Indonesia yang mendukung
pembelajaran bahasa Indonesia SD/MI 
SUB KOMPETENSI 
1.4 Mengkreasikan apresiasi sastra Indonesia yang mendukung pembelajaran bahasa
Indonesia 
INDIKATOR ESENSIAL 
1.4.1 Menganalisis unsur intrinsik dan ekstrinsik, stuktur, dan ciri-ciri karya sastra puisi 
1.4.2 Menganalisis unsur intrinsik dan ekstrinsik, stuktur, dan ciri-ciri karya sastra prosa 
1.4.3 Menyusun langkah-langkah membuat paraphrase puisi ke prosa 
1.4.4 Menilai Prosa 
1.4.5 Mengapresiasi Drama 

CONTOH SOAL 
No Soal 15a (sesuai IE 1.4.1) 
Berikut ini termasuk unsur intrinsik dalam puisi kecuali… 
a. Tema 
b. Amanat 
c. Tokoh 
d. Gaya bahasa 
Jawaban C 
Pembahasan 
Unsur intrinsik sebuah puisi terdiri dari tema, amanat, sikap atau nada, perasaan, tipografi,
enjambemen, akulirik, rima, citraan, dan gaya bahasa 

No Soal 15b (sesuai IE 1.4.1) 


Berikut ini termasuk unsur ektrinsik dalam puisi kecuali… 
a. Latar belakang penciptaan 
b. Kondisi masyarakat pada saat karya sastra diciptakan 
c. Pandangan hidup pengarang 
d. Pembuatan alur yang sesuai 
Jawaban D 
Pembahasan Unsur Ekstrinsik puisi adalah 
1. Latar belakang penciptaan 
2. Kondisi masyarakat pada saat karya sastra diciptakan 
3. Pandangan hidup pengarang 

No Soal 15c (sesuai IE 1.4.1) 


Berdasarkan kemunculan waktunya, pantun termasuk jenis puisi… 
a. Puisi lama 
b. Puisi baru 
c. Puisi modern 
d. Puisi ortodok 
Jawaban A 
Pembahasan 
Berdasarkan waktu kemunculannya puisi dapat dibagi menjadi 3 kelompok, yaitu puisi
lama, puisi baru, dan puisi modern. 
1. Puisi lama adalah puisi yang lahir sebelum masa penjajahan Belanda, sehingga belum
tampak adanya pengaruh dari kebudayaan barat. Sifat masyarakat lama yang statis dan
objektif, melahirkan bentuk puisi yang statis pula, yaitu sangat terikat pada aturan
tertentu. Puisi lama terdiri dari mantra, bidal, pantun dan karmina, talibun, seloka,
gurindam, dan syair. 
2. Puisi baru adalah puisi yang muncul pada masa penjajahan Belanda, sehingga pada puisi
baru tampak adanya pengaruh dari kebudayaan Eropa. Penetapan jenis puisi baru
berdasarkan pada jumlah larik yang terdapat dalam setiap bait. Jenis puisi baru dibagi
menjadi distichon, terzina, quatrain, quint, sextet, septima, stanza atau oktaf, serta soneta. 
3. Puisi modern adalah puisi yang berkembang di Indonesia setelah masa penjajahan
Belanda. Berdasarkan cara pengungkapannya, puisi modern dapat dibagi menjadi puisi
epik, puisi lirik, dan puisi dramatik. 

No Soal 16a (sesuai IE 1.4.2) 


Berikut ini termasuk unsur intrinsik prosa kecuali 
a. Tema dan amanat 
b. Tokoh 
c. Sudut pandang 
d. Sejarah pengarang 
Jawaban D 
Pembahasan 
Unsur intrinsik prosa terdiri dari tema dan amanat, alur, tokoh, latar, sudut pandang, serta
bahasa yang dipergunakan pengarang untuk mengekspresikan gagasannya 

No Soal 16b (sesuai IE 1.4.2) 


Berikut ini contoh prosa yang termasuk sastra klasik… 
a. Roman 
b. Fable 
c. Legenda 
d. Hikayat 
Jawaban A 
Pembahasan 
Sastra modern termasuk di dalamnya prosa baru yang mencakup roman, novel, novel
populer, cerpen. Sastra klasik termasuk di dalamnya yaitu prosa lama yang mencakup
cerita rakyat, dongeng, fabel, epos, legenda, mite, cerita jenaka, cerita pelipur lara, sage,
hikayat, dan silsilah. 

No Soal 16c (sesuai IE 1.4.2) 


Alur dimana mulai terjadinya pertentangan antar pelaku terjadi pada tahap … 
a. Ekposisi 
b. Konflik 
c. Komplikasi 
d. Klimaks 
Jawaban B 
Pembahasan 
TAHAP-TAHAP ALUR 
1. Tahap perkenalan/Eksposisi adalah tahap permulaan suatu cerita yang dimulai dengan
suatu kejadian, tetapi belum ada ketegangan (perkenalan para tokoh, reaksi antarpelaku,
penggambaran fisik, penggambaran tempat) 
2. Tahap pertentangan /Konflik adalah tahap dimana mulai terjadi pertentangan antara
pelaku-pelaku (titik pijak menuju pertentangan selanjutnya) 
3. Tahap penanjakan konflik/Komplikasi adalah tahap dimana ketegangan mulai terasa
semakin berkembang dan rumit (nasib pelaku semakin sulit diduga, serba samar-samar) 
4. Tahap klimaks adalah tahap dimana ketegangan mulai memuncak (perubahan nasip
pelaku sudah mulai dapat diduga, kadang dugaan itu tidak terbukti pada akhir cerita) 
5. Tahap penyelesaian adalah tahap akhir cerita, pada bagian ini berisi penjelasan tentang
nasib-nasib yang dialami tokohnya setelah mengalami peristiwa puncak itu. Ada pula yang
penyelesaiannya diserahkan kepada pembaca, jadi akhir ceritanya menggantung, tanpa ada
penyelesaian. 

No Soal 16d (sesuai IE 1.4.2) 


Alur sebuah prosa dimana cerita yang disampaikan diutarakan mulai dari awal sampai
masa kini dan masa yang akan datang dinamakan 
a. Alur maju 
b. Alur mundur 
c. Alur campuran 
d. Alur mengambang 
Jawaban A 
Pembahasan 
MACAM-MACAM ALUR 
1. Alur maju adalah peristiwa –peristiwa diutarakan mulai awal sampai akhir/masa kini
menuju masa datang. 2. Alur mundur/Sorot balik/Flash back adalah peristiwa-peristiwa
yang menjadi bagian penutup diutarakan terlebih dahulu/masa kini, baru menceritakan
peristiwa-peristiwa pokok melalui kenangan/masa lalu salah satu tokoh. 
3. Alur gabungan/Campuran adalah peristiwa-peristiwa pokok diutarakan. Dalam
pengutararaan peristiwa-peristiwa pokok, pembaca diajak mengenang peristiwa-peristiwa
yang lampau,kemudian mengenang peristiwa pokok ( dialami oleh tokoh utama) lagi. 
No Soal 17a (sesuai IE 1.4.3) 
Mengubah sebuah puisi menjadi prosa dengan cara menambah sejumlah kata pada puisi
sehingga kalimat dalam puisi mudah dipahami yaitu ciri dari… 
a. Parafrase terikat 
b. Parafrase bebas 
c. Parafrase maju 
d. Parafrase mundur 
Jawaban A 
Pembahasan 
Jenis Parafrase 
a. Parafrase terikat, yaitu mengubah puisi menjadi prosa dengan cara menambahkan
sejumlah kata pada puisi sehingga kalimat-kalimat puisi mudah dipahami. Seluruh kata
dalam puisi masih tetap digunakan dalam parafrase tersebut. 
b. Parafrase bebas, yaitu mengubah puisi menjadi prosa dengan kata-kata sendiri. Kata-
kata yang terdapat dalam puisi dapat digunakan, dapat pula tidak digunakan. Setelah kita
membaca puisi tersebut kita menafsirkan secara keseluruhan, kemudian menceritakan
kembali dengan kata-kata sendiri. 

No Soal 17b (sesuai IE 1.4.3) 


Langkah-langkah membuat parafrase prosa dari puisi yaitu 
1. Menulis ulang puisi dalam bentuk prosa 
2. Baca puisi berulang-ulang hingga paham 
3. Tambahkan kata atau tanda baca sesuai dengan pemahaman dengan tanda kurung 
Urutan yang benar dalam membuat parafrase puisi menjadi prosa adalah 
a. 1-2-3 
b. 2-3-1 
c. 2-1-3 
d. 1-3-2 
Jawaban C 
Pembahasan lihat dan telaah jawaban 

No Soal 18 (sesuai IE 1.4.4) 


Pada suatu sore, datanglah tiga anak kecil ke Salemba dalam langkah malu-malu. Mereka
menyerahkan sebuah karangan bunga yang berpita hitam. Karangan bunga itu diserahkan
sebagai tanda ikut berduka cita terhadap kakak mereka (orang yang mereka anggap
kakak), yang telah ditembak mati pada siang hari itu. 
Sudut pandang prose diatas adalah 
a. Orang pertama 
b. Orang kedua 
c. Orang ketiga 
d. Orang keempat 
Jawaban C 
Pembahasan 
Sudut pandang suatu sastra dikenal dengan 2 sudut pandang 
1. Sudut pandang orang pertama yaitu menceritakan suatu cerita sehingga penulis seperti
menulis tentang dirinya dan biasanya banyak kata Aku 
2. Sudut pandang orang ketiga yaitu penulis menceritakan seseorang dalam tulisannya dan
biasanya disebut suatu nama atau kata ungkapan lain seperti mereka. 

No Soal 19 (sesuai IE 1.4.5) 


Pendekatan yang dipakai untuk mengapresiasi sebuah derama yaitu dengan
membandingkat tiap-tiap kesatuan pristiwa sehingga sampai pada suatu kesimpulan
bagaimana citra atau ide yang hendak disampaikan disebut… 
a. Pendekatan segi fungsi 
b. Pendekatan derajat peristiwa 
c. Pendekatan terhadap tema 
d. Pendekatan dari sudut gaya 
Jawaban C 
Pembahasan 
Berdasarkan teori-teori yang dijelaskan sebelumnya untuk mengapresiasi sastra drama,
ada beberapa pendapat yang dapat dilakukan untuk mengapresiasi sastra drama. Menurut
Hamidy (1984:15) pendekatan tersebut dapat dilakukan dalam segi: 1. Pendekatan dari
segi fungsi. Hal ini biasanya dihubungkan dengan peranan yang dapat dimainkan oleh
drama dalam masyarakat. 2. Pendekatan derajat peristiwa. Pembahasan ini berhubungan
dengan alur, yaitu dalam bentuk bagaimana derajat peristiwa seperti eksposisi, komplikasi,
krisis, sampai kepada penyelesaian. 3. Pendekatan terhadap tema. Dalam hal ini kita
dihadapkan kepada perbandingan tiap-tiap kesatuan peristiwa sehingga sampai kepada
suatu logika (kesimpulan) bagaimana citra atau ide yang hendak disampaikan. 4.
Pendekatan terhadap drama yang berkaitan dengan segi aliran karya sastra, misalnya
realisme, naturalisme, dan ekspresionisme. 5. Pendekatan dari sudut gaya. Pembahasan ini
menyangkut bagaimana perkembangan sistematika bangun drama itu dengan kaitannya
terhadap pantulan gaya yang hendak diperlihatkan kepada pembaca. 

Anda mungkin juga menyukai